TSRA Clinical Scenarios - Adult Cardiac Surgery Flashcards

1
Q

Heparin dose for CPB?

A

400 units/kg

How well did you know this?
1
Not at all
2
3
4
5
Perfectly
2
Q

ACT required for initiation of CPB?

A

400-480 (480 is book answer).

How well did you know this?
1
Not at all
2
3
4
5
Perfectly
3
Q

5 min after cross clamp is released when coming off CPB, the heart begins to distend w/o contraction.
What are the general causes of this situation?
What do you do first?
What if this doesn’t work?

A

Two possible problems:
1) valve leak
2) heart is not ejecting

Don’t let the myocardium blow up:
1) Squeeze the heart.
2) Pace the heart.
3) If this does not resolve quickly, cross clamp, go up on bypass, and vent.
4) Talk to anesthesia and perfusion to see if they can help determine a cause on echo or via labs.

How well did you know this?
1
Not at all
2
3
4
5
Perfectly
4
Q

Coming off CPB, you attempt to pace, but have no capture.
You check the leads - they are well placed.
You change the box and wires - same problem.
You ask perfusion what K is - 7.
You ask anes what UOP is - minimal.

You cross clamp and vent the heart.
How do you manage K?

A

Use the pump: Hemoconcentrate.
Use anesthesia: IV insulin with glucose. Bicarb and/or lasix if there is UOP.
Be careful of hyperkalemia. You will lose the ability to pace, this pertains to postop as well.

*You have to re-clamp and vent because the heart is not ejecting.
*Perfusion/anesthesia should have been keeping track of the K.

How well did you know this?
1
Not at all
2
3
4
5
Perfectly
5
Q

A patient fibrillates when coming off CPB.
How do you defibrillate?
What if first couple defib attempts don’t work?

What if distended?

What other adjuncts can be used/what needs to be checked and optimized?

A

Internal paddles set to 10-20 joules and gradually increased.
If not working, give IV lidocaine 1 mg/kg and IV amiodarone bolus 150 and try again.

If distended, manually squeeze the heart and empty with CPB, place vents if needed, then cardiovert.

1) Vent air in root.
2) Inc MAP to 75 (coronary perfusion and to flush out embolized air).
3) Optimize O2, lytes, and temp.
4) Check Echo for AI.

How well did you know this?
1
Not at all
2
3
4
5
Perfectly
6
Q

If heart is distending when coming of CPB, what amount of AI requires consideration for replacement?

A

AI greater than 1+/moderate-severe requires exploration and replacement of the aorta.
Should be higher on differential after a mitral surgery - damage to noncoronary leaflet w/ placement of the mitral stitches.

How well did you know this?
1
Not at all
2
3
4
5
Perfectly
7
Q

Explain CPB components.

A

Venous cannulas drain to reservoir by gravity or vacuum, pumped into the oxygenator/heat exchanger, then continues to arterial air filter and to the arterial cannula in the patient.

How well did you know this?
1
Not at all
2
3
4
5
Perfectly
8
Q

Attempt CPB, but perfusionist says high aortic line pressure. What is differential?

A

Obstruction - kink or clamp
Malposition - in one of the aortic branches
Cannula too small - 21-24 Fr should be adequate (18-20 used at USC)
Aortic dissection - systemic P low w/ abnormal ascending aorta

How well did you know this?
1
Not at all
2
3
4
5
Perfectly
9
Q

What is your differential and initial troubleshooting for inadequate venous drainage on CPB (ie heart is full)?

What if the heart is empty, but reservoir is empty also?

A

Try circuit issues first - check for air lock and cannula malposition.
Is there chattering?
If so, reduce flows if flow is high.
If not, may need to increase cannula size or just add suction.

Make sure no other avenues of blood flow into the heart - AI needs an LV vent, azygous vein needs adjustment to snares to exclude them, L SVC needs snare or cannulation depending on situation (cannula if NO innominate).

If heart is empty and so is the reservoir can add volume.
If persistent after replacement, consider retroperitoneal or peritoneal hemorrhage. Do an abdominal exam.

How well did you know this?
1
Not at all
2
3
4
5
Perfectly
10
Q

Attempting CPB flows, but MAP not rising above 40. Pt has hx of ACE use.

A

Rule out other problems. Could be vasoplegia.
Give pressors - phenylephrine, norepi, vasopressin. Methylene blue can be used as well.

This can occur postop as well.

How well did you know this?
1
Not at all
2
3
4
5
Perfectly
11
Q

You place retrograde cardioplegia line in coronary sinus, but pressures are low. How do you troubleshoot?

A

Check for rupture of the sinus - inspect inferior aspect of the heart
Check position of cannula
Check if balloon is ruptured

Persistent L SVC

How well did you know this?
1
Not at all
2
3
4
5
Perfectly
12
Q

When giving retrograde cardioplegia, you find a L SVC. How do you manage?

A

Check if there is an innominate.
Present - can snare below innominate drainage to include L SVC in circuit.
Not present - cannulate the L SVC and add it to venous drainage.

How well did you know this?
1
Not at all
2
3
4
5
Perfectly
13
Q

Coming off CPB from a mitral surgery. BP drops, ST elevation on EKG, and RV distends. You suspect air in the coronary.
Why does this happen?
How do you manage? What are the adjuncts?

A

R coronary ostium is anterior and susceptible to air embolism.
Re-institute CPB, empty the heart, and flow w/ high perfusion pressure (MAP 70-75) to help support cardiac fct and push air through coronary into venous circulation.
Add a root vent to prevent further air migration into the coronary arteries.
A needle in the apex of the heart can be added to remove air if too much in the apex.

How well did you know this?
1
Not at all
2
3
4
5
Perfectly
14
Q

If patient has previous CABG and is requiring CPB, how do you manage an open LIMA?

A

Consider retrograde.
Can clamp vs cold bypass flow.

How well did you know this?
1
Not at all
2
3
4
5
Perfectly
15
Q

If patient has open SVGs after CABG and needs CPB, but the grafts are high on the aorta, what is an option to make room for cross clamp?

A

Axillary cannulation may be needed to make room for an aortic cross clamp.

How well did you know this?
1
Not at all
2
3
4
5
Perfectly
16
Q

In a redo patient w/ previous CABG w/ open LIMA, what is the problem with antegrade cardioplegia? What are possible solutions?

A

Antegrade will perfuse the OM and PDA territories, but not the LAD.
Retrograde will perfuse all. Ideally, the LIMA is found and clamped. If not, the heart needs to be cooled very well, and intermittent retrograde perfusion may need to be given.

How well did you know this?
1
Not at all
2
3
4
5
Perfectly
17
Q

In the setting of aortic insufficiency, discuss options for CPB.

A

Question is how to deliver plegia.

Retrograde cardioplegia would ensure delivery. Antegrade would likely require direct delivery of cardioplegia.

How well did you know this?
1
Not at all
2
3
4
5
Perfectly
18
Q

How could you incompletely deliver cardioplegia if using retrograde cardioplegia assuming no L SCV? What can you do?

A

Tip of the retrograde cannula is distal to the middle cardiac vein. Direct retrograde insertion can help (bicaval cannulation, snare cavas, and open R atrium).

How well did you know this?
1
Not at all
2
3
4
5
Perfectly
19
Q

What can make retrograde cardioplegia delivery riskier in a redo operation (previous CABG)?

A

If you perforate the sinus, the posterior heart may be stuck, making dissecting in a bloody field more difficult.
Also, the anterior/lateral cardiac wall dissections on the front end may damage CABGs.

How well did you know this?
1
Not at all
2
3
4
5
Perfectly
20
Q

After cross clamping, 500 ml of antegrade cardioplegia is delivered, but there is poor distention of the aortic root, incomplete arrest, and LV distention.

What is happening and how do you manage?

A

Aortic insufficiency. Should not have been a surprise.

Switch to retrograde cardioplegia and turn on the aortic root vent vs an LV vent to decompress the heart.
Can also do direct cardioplegia if operating on aorta or aortic valve anyway (sounds like you may need to).

How well did you know this?
1
Not at all
2
3
4
5
Perfectly
21
Q

How do you confirm adequacy of retrograde cardioplegia catheter delivery?
What’s the appropriate pressure for cannula flow?

A

Cessation of electrical and myocardial activity.
Appropriate pressure in cannula is around 40 mmHg.
Observe flow through coronary veins and arteries. Can also check coronary ostia through aortotomy for backflow.
Finally, heart should be cooling - check back, front, and lateral.

How well did you know this?
1
Not at all
2
3
4
5
Perfectly
22
Q

What is a concern with retrograde cannula placement even if it works well to arrest the heart?

A

Myocardial edema. This is why the balloons on the cannula are not usually 100% occlusive.

How well did you know this?
1
Not at all
2
3
4
5
Perfectly
23
Q

Where does a persistent L SCV usually drain?

A

Into the coronary sinus, may partially drain via innominate V.

How well did you know this?
1
Not at all
2
3
4
5
Perfectly
24
Q

You attempt to infuse antegrade cardioplegia. The perfusionist notes high line pressure. How do you manage?

A

Stop flow. Look for kinks or clamps to r/o obstruction. Ensure pressure monitoring line is connected.
If all above ruled out, concern for dissection. Visualize the root and ask anes to look at aorta.
If dissection, cool with original aorta cannula distal to the clamp and prepare for dissection repair.

How well did you know this?
1
Not at all
2
3
4
5
Perfectly
25
Q

After a cardiac surgery pump case w/ retrograde cardioplegia, the myocardium is slow to regain activity. What should you check?

A

Check retrograde catheter and/or that the balloon is down, in order to allow for adequate coronary sinus flow.

How well did you know this?
1
Not at all
2
3
4
5
Perfectly
26
Q

When sewing a CABG, you notice bleeding from the arteriotomy. What does this mean?

A

Arrested myocardium is getting perfusion. Check the cross clamp and that the root vent is on.
There could be collaterals.
If unable to ID cause, monitor for electrical activity, and consider cold topical saline, cooling the patient to mild hypothermia (32 C), reducing flows as tolerated, or increasing frequency of cardioplegia administration.

Can deliver some plegia through vein graft once suturing complete.

How well did you know this?
1
Not at all
2
3
4
5
Perfectly
27
Q

How do you manage a perforation of the coronary sinus after placement of a retrograde cardioplegia catheter?

A

CPB, clamp, and arrest with antegrade if possible or direct if retrograde was because of inability to do antegrade.
Repair the hole directly with Prolene or a pericardial patch.

How well did you know this?
1
Not at all
2
3
4
5
Perfectly
28
Q

What are the three broad categories for etiology of persistent electrocardial activity (problem with initiating plegia in CPB)?

A

Access - plegia cannula placement, kinking
Collateral - clamp not across, drainage poor, L SVC, vents/suckers; hypothermia?
Myocardial mass - hypertrophy may require more plegia or more hypothermia

How well did you know this?
1
Not at all
2
3
4
5
Perfectly
29
Q

Redo sternotomy, and there is a significant amount of dark blood from the sternomanubrial jct. What was injured?

A

Innominate vein.

How well did you know this?
1
Not at all
2
3
4
5
Perfectly
30
Q

Redo mitral w/ innominate vein injury during sternotomy. How do you manage?
How do you get on bypass?
What are some options if first surgical attempt at the injury fails?
What can be done if more exposure needed?

A

Pack the area and close the sternum with penetrating towel clamps.
Peripheral CPB. Put pump suckers in the injured field.
Reopen and dissect vein length for assessment and attempt at closure w/o tension. Primary vs pericardial patch repair.
If cannot repair, divide and oversew.
If too lateral, may need trapdoor to expose - above clavicle and at 3rd rib space.

How well did you know this?
1
Not at all
2
3
4
5
Perfectly
31
Q

Redo case. You attempt to dissect SVC for cannulation, but make a large hole before pursestring is in. How do you manage?
What is the first step?
How do you cannulate?
How do you clear field?
How do you manage an azygous contributing to bleed?

A

Tamponade it.
Attempt IVC and aortic cannulation for CPB. Need vacuum venous drainage to prevent air lock.
Pump suckers near SVC to clear the field.
May be possible to cannulate through the injury.
Consider placing pump sucker into azygous vs ligating azygous if you can’t see.
May need to dissect more proximal and place a Rummel tourniquet vs vascular clamps.
Once controlled, may need primary vs pericardial patch.

Be prepared for phrenic nerve injury with all the dissection around the SVC.

How well did you know this?
1
Not at all
2
3
4
5
Perfectly
32
Q

You are called to cath lab during a lead extraction after the patient arrested w/ hypotension.
What injury is likely? IE what structure?
How do you manage?

A

Venous injury, often in the SVC.
Cannulate the groins and start CPB vs open to relieve tamponade and compress SVC while achieving CPB.
Open and decompress the heart. Identify the injury, dissect proximal and distal, control it, and repair primarily vs w/ patch.

How well did you know this?
1
Not at all
2
3
4
5
Perfectly
33
Q

A redo cardiac surgery patient is femorally cannulated with difficulty during the venous cannulation. The venous line has return, but the patient continues to become hypotensive after CPB is started.
What should be your suspicion?
How do you diagnose?
Does management change if chest is open?
What if the injury is high?

A

Femoral venous injury - could be retroperitoneal (likely iliac) or intra-abdominal.
Goals: identify injury, cannulate beyond repair.

Venogram if available.

Chest is open: can centrally cannulate, otherwise, may need access in contralateral femoral vein.

Injury is above iliacs: use fluoro to get wire and cannula above and go on, then open the abdomen and repair the injury.

How well did you know this?
1
Not at all
2
3
4
5
Perfectly
34
Q

SVC is ruptured during balloon dilation for SVC syndrome. How do you manage?
What’s your cannulation strategy?

A

Give heparin.
Emergency median sternotomy.
Bicaval cannulate - for the SVC, enter the RA and traverse cannula through injury - will have to use a malleable cannula, not a right angle.
Empty the heart.
Primary pericardial patch closure.

How well did you know this?
1
Not at all
2
3
4
5
Perfectly
35
Q

You’re trying to get bicaval cannulation in an old lady, and the IVC tears low near the diaphragm.
How do you manage? What are your goals?
How do you get more exposure?

A

Initial goals: get venous access and go on CPB.
If possible, traverse the IVC cannula through the injury from above. May need a malleable cannula for this.

Next goal: get control below the injury or reduce blood flow to the injury.
Try to place a Rummel clamp below the injury.
If unable - SVC cannula for everything above, suckers to control bleeding in the pericardial well. Get femoral vein to control bleeding from below. Be careful with distance of femoral cannula is to not worsen the injury.

Can attempt incising the diaphragm through sternotomy to get exposure.
If unable - extend sternotomy inferiorly and convert to RUQ subcostal incision.

How well did you know this?
1
Not at all
2
3
4
5
Perfectly
36
Q

Cath lab creates an RV puncture and places a pericardial drain w/ bloody drainage. Manage.

A

Heparin and sternotomy.
CPB and arrest the heart.
Wide patch repair even if small.
Bypass the coronary lesion if during a cath case for coronary disease.

How well did you know this?
1
Not at all
2
3
4
5
Perfectly
37
Q

There is a small R PA injury noticed after an aortic surgery when coming off bypass. Manage.

A

If small, a figure of 8 will fix.
If large or uncontrolled, go back on bypass (if you had axillary, can do end to end of previous graft).
Decompress the heart, and the PA will decompress.
Mobilize the R PA. May need pericardial patch.

How well did you know this?
1
Not at all
2
3
4
5
Perfectly
38
Q

You puncture the opposite side of the aorta during arterial cannulation. Manage.

A

May need circulatory arrest.
Can cool with the existing cannula if forward flow is okay.
May need to establish axillary access.
Small aortotomy and local repair.

How well did you know this?
1
Not at all
2
3
4
5
Perfectly
39
Q

What symptoms represent unstable angina?

A

chest pain at rest, exertional angina not relieved by rest, and new onset of chest pain

How well did you know this?
1
Not at all
2
3
4
5
Perfectly
40
Q

What symptoms represent stable ischemic heart disease

A

exertional angina - predictable, improves with rest

How well did you know this?
1
Not at all
2
3
4
5
Perfectly
41
Q

What is included in CAD guideline-directed medical therapy?

A

lifestyle modification, statin therapy (LDL goal 70-100), beta-blockade, aspirin, and ACE inhibition (if LV dysfunction, DM, or CKD)

How well did you know this?
1
Not at all
2
3
4
5
Perfectly
42
Q

What are hemodynamically significant coronary lesions on left heart cath?

A

Left main >50%
non-LM >70%

How well did you know this?
1
Not at all
2
3
4
5
Perfectly
43
Q

What FFR is hemodynamically significant? When is FFR used?

A

< 0.80.
Determines hemodynamic significance of the cumulative effect of proximal stenosis.
Used to guide revasc in angiographically intermediate coronary stenosis in patients w/ stable angina.

How well did you know this?
1
Not at all
2
3
4
5
Perfectly
44
Q

What is the SYNTAX score?

A

16 segments in the coronary tree.
Lesions in this tree are scored.
The sum of these scores is the overall SYNTAX score.

How well did you know this?
1
Not at all
2
3
4
5
Perfectly
45
Q

What are the cutoffs for the SYNTAX score in terms of complexity?
What was the main implication of the SYNTAX trial?

A

> 22 is intermediate complexity.
32 is high complexity.
The results of the SYNTAX trial revealed that patients with 3-vessel (70%) or L main (50%) disease benefited from CABG over PCI.

How well did you know this?
1
Not at all
2
3
4
5
Perfectly
46
Q

What is a non-invasive option for imaging the coronary arteries?

A

Gated cardiac CT

How well did you know this?
1
Not at all
2
3
4
5
Perfectly
47
Q

What did a secondary analysis of the STITCH trial reveal concerning myocardial viability studies?

A

They are obtained in preop CABG pts w/ EF <35%. They can predict an improvement in EF postoperatively, but this did not translate into a survival benefit. IE it should not be used as a way to rule out low EF patients for CABG over medical therapy.

How well did you know this?
1
Not at all
2
3
4
5
Perfectly
48
Q

How do you decide who needs coronary revascularization in patients with stable ischemic heart disease?

A

Activity limiting symptoms despite maximal medical therapy.
Active patients who want PCI for better QoL compared to med therapy.
Anatomy with proven survival benefit.

How well did you know this?
1
Not at all
2
3
4
5
Perfectly
49
Q

After deciding on revascularization, how do you decide who needs CABG?

A

1) 3v disease, especially w/ syntax >22 (mod) and low surgical risk.
2) Significant LM disease (50%).
3) Multivessel (>1) w/ either proximal LAD disease, or moderate/high complexity anatomy, or diabetes.
3a) Alternatively, pts w/ relatively well preserved LVEF, low complexity coronary anatomy, and no diabetes - CABG is reasonable.
4) For single vessel disease, pts for whom it is reasonable to prefer CABG include those with a large amount of potentially ischemic myocardium in the LAD distribution, those who are not candidates for long-term DAPT, or those who have failed prior PCI. Patients with ischemic cardiac arrest and anatomically complex disease may also be considered for CABG.

I.E. Patients with single-vessel not meeting criteria above OR two-vessel disease involving the RCA and L Cx coronary arteries may reasonably choose PCI.

Europe guidelines add:
Multivessel (2+) w/ EF <40.
Multivessel (2+) w/ >10% ischemic territory.

How well did you know this?
1
Not at all
2
3
4
5
Perfectly
50
Q

Management of significant CAD in 1 vessel w/ refractory angina despite medical therapy and PCI.

A

CABG.

How well did you know this?
1
Not at all
2
3
4
5
Perfectly
51
Q

Management of significant CAD in 1 vessel after sudden cardiac arrest from ischemic ventricular arrhythmia.

A

CABG.

How well did you know this?
1
Not at all
2
3
4
5
Perfectly
52
Q

Management of CAD at 50% in 1 vessel in patients undergoing valve or aortic surgery?

A

Concurrent CABG.

How well did you know this?
1
Not at all
2
3
4
5
Perfectly
53
Q

What is the ideal graft for the LAD?

A

IMA. Left preferred. Right is second choice.

How well did you know this?
1
Not at all
2
3
4
5
Perfectly
54
Q

In a patient undergoing CABG for 3 vessels without excessive risk of sternal complications or other organ failure, what are the ideal grafts?

A

BIMA and radial.

How well did you know this?
1
Not at all
2
3
4
5
Perfectly
55
Q

What can be done technically to reduce the risk of sternal complications in a patient getting BIMA grafts?

A

Skeletonized.

How well did you know this?
1
Not at all
2
3
4
5
Perfectly
56
Q

Known risks of sternal infection and malunion in cardiac surgery?

A

Nonelective procedure, age, uncontrolled DM (HbA1c >7), BMI >40, female, COPD, preop hospitalization >3 days, smoking, immunosuppression regimen, radiation mediastinal injury

How well did you know this?
1
Not at all
2
3
4
5
Perfectly
57
Q

When should CABG patients receive aspirin?

A

Preoperatively when they discover CAD is ideal.
Within 6 hrs postop after CABG.
Continued indefinitely.

How well did you know this?
1
Not at all
2
3
4
5
Perfectly
58
Q

A patient comes in for elective CABG but has not stopped taking plavix. What do you do?

A

Cancel the case.
Plavix and ticagrelor (Brilinta) need to be stopped for 5 days preop.

Aspirin, on the other hand, can be continued up to the day of CABG and resumed after.

How well did you know this?
1
Not at all
2
3
4
5
Perfectly
59
Q

A patient requires an urgent CABG but is taking plavix.
What do you do?

A

Ideally, P2Y12 inh should be stopped 5 days preop.
In an urgent setting, holding for 24 hrs preop will reduce major bleeding.

Short acting IV antiplatelets (eptifibatide [Integrillin] or tirofiban [Aggrastat]; both gpIIb/IIIa inh) should stopped 2-4 hrs preop.
Abciximab (reopro) should be stopped 12 hrs preop.

How well did you know this?
1
Not at all
2
3
4
5
Perfectly
60
Q

Why are beta blockers reinstituted after CABG (as long as no contraindications)?

A

Reduce incidence and sequela of atrial fibrillation; of note, preoperative beta blockade does not seem to help.

How well did you know this?
1
Not at all
2
3
4
5
Perfectly
61
Q

What is the target for statin therapy in patients undergoing CABG?

A

reduce LDL <100 and achieve at least 30% dec in LDL

How well did you know this?
1
Not at all
2
3
4
5
Perfectly
62
Q

Which postop CABG patients require ACE or ARB?

A

LVEF <40, HTN, DM, CKD. Unless otherwise contraindicated.
Almost all of them.

How well did you know this?
1
Not at all
2
3
4
5
Perfectly
63
Q

What is the continuous IV insulin target in postop CABG?

A

blood glucose concentration <180

How well did you know this?
1
Not at all
2
3
4
5
Perfectly
64
Q

When anatomically and clinically suitable, what is the RIMA often used for?

A

L Cx or RCA usually if critically stenosed and perfusing LV myocardium - can improve survival and decrease reintervention

How well did you know this?
1
Not at all
2
3
4
5
Perfectly
65
Q

RIMA patency is directly related to what?

A

Degree of proximal stenosis of the target vessel. Ie how much competitive flow is there?

How well did you know this?
1
Not at all
2
3
4
5
Perfectly
66
Q

What lesions should a radial artery be used for?
What is patency of the RA prone to?
What specific postop medication improves patency?

A

Prone to spasm in periop period and sensitive to competitive flow.
Only use for L side lesions >70% (severe) or R side lesions >90% (critical) - same for gastroepiploic arteries.
Give ca channel blocker postop.

Total arterial bypass is ideal.

How well did you know this?
1
Not at all
2
3
4
5
Perfectly
67
Q

Who is the ideal patient who benefits most from complete arterial revascularization?

A

<60, with few comorbidities (no DM, ESRD), severe L sided stenosis, and critical (90%) R sided stenosis (radials work better here).
Skeletonize.

How well did you know this?
1
Not at all
2
3
4
5
Perfectly
68
Q

What are inferior leads?

A

II, III, aVF

How well did you know this?
1
Not at all
2
3
4
5
Perfectly
69
Q

What EKG findings suggests RV or RCA ischemia (or L dominant PDA disease)?

A

Inferior leads: II, III, aVF.
Or posterior findings: reciprocal V1-2.

How well did you know this?
1
Not at all
2
3
4
5
Perfectly
70
Q

What EKG changes suggest LV or LAD/LCx territory ischemia?
There are 3 associated territories, what are they, and what leads indicate each?

A

Anteroseptal: V1-V2.
Anteroapical: V3-4.
Anterolateral: V5-6, I, aVL.

How well did you know this?
1
Not at all
2
3
4
5
Perfectly
71
Q

What is diagnostic for STEMI?

A

Angina symptoms for 20 min w/ ST elevation 2+mm in 2 contiguous leads or new LBBB. Greater risk for transmural ischemia.

How well did you know this?
1
Not at all
2
3
4
5
Perfectly
72
Q

What is the diagnostic criteria for NSTEMI?

A

Angina symptoms >10 mins.
Elevated cardiac biomarkers.
ST elevation of 0.5-1mm
- or ST depression >0.5 mm
- or T wave inversion >1mm
More likely to have subendocardial ischemia.

How well did you know this?
1
Not at all
2
3
4
5
Perfectly
73
Q

When can IABP be useful in the setting of MI (3 situations listed)?

A

Refractory shock despite initial medical management; post-infarct VSD; acute papillary muscle rupture w/ MR.
Often as a very temporary bridge. If the heart’s viability is significantly in question, then an impella may be a better choice as long as the RV works. If biventricular failure, then ECMO would probably be a better choice (may still need Impella or IABP or pressors to empty LV).

How well did you know this?
1
Not at all
2
3
4
5
Perfectly
74
Q

What are contraindications for IABP?

A

Severe AI and PVD precluding placement.

How well did you know this?
1
Not at all
2
3
4
5
Perfectly
75
Q

What is the first line reperfusion strategy for STEMI?
Timing?

A

PCI. Door to balloon time of 90 min.

How well did you know this?
1
Not at all
2
3
4
5
Perfectly
76
Q

What is the role for CABG in STEMI?

A

Failure of PCI + coronary anatomy suitable + persistent ischemia.
OR MI complication - papillary muscle rupture, post-infarct VSD, LV rupture

How well did you know this?
1
Not at all
2
3
4
5
Perfectly
77
Q

What coronary disease is left-main equivalent?

A

> 70% in LAD and L Cx

How well did you know this?
1
Not at all
2
3
4
5
Perfectly
78
Q

A patient presents w/ NSTEMI, stable, meets indication for CABG. How do you manage?

A

Maximize medical mgmt: heparin, ASA, O2 if needed, IABP if refractory angina or low EF.
Urgent CABG (plavix should be stopped 24 hrs).
The patient should not be DC home.

How well did you know this?
1
Not at all
2
3
4
5
Perfectly
79
Q

A patient is resuscitated from witnessed cardiac arrest. Coronary angiography shows 3V disease. What’s the revascularization strategy?

A

PCI of the INFARCT artery. THEN assess if cardiogenic shock. THEN assess for area of infarction.
- If stable w/ low complexity - PCI of non-culprit arteries.
- If cardiogenic shock - DON’T revasc non-culprit arteries. DEFER this… and determine at-risk myocardium.

Non-culprit supplies large area of myocardium?
No -> GDMT and heart team discussion.
Yes -> Complex/multivessel?
- No -> staged PCI.
- Yes -> heart team: GDMT w/ staged PCIs vs CABG

How well did you know this?
1
Not at all
2
3
4
5
Perfectly
80
Q

Manage pt w/ inferior infarction and RV involvement. PCI failed.

A

CABG.
Consider delaying until RV is optimized w/ inotropes, diuretics, or even mechanical support.

How well did you know this?
1
Not at all
2
3
4
5
Perfectly
81
Q

A patient w/ previous coronary stent undergoes CABG. How do you manage this vessel?

A

If it is wide open, no reason to bypass as it will likely fail 2/2 competitive flow.

How well did you know this?
1
Not at all
2
3
4
5
Perfectly
82
Q

A patient requires CABG and is found to have a porcelain aorta. How does this affect planning?
Preop eval?
Cannulation?
Grafts?
Proximal placement if vein grafts?
Thoughts on clamping?

A

Preop: evaluate groins and axilla w/ CT.
Consider beating heart CABG - may need fluids, pressors, and shunt.
If needed, peripheral cannulation: axillary or femoral.
Try to use all arterials, and Y off the IMAs if needed.
Where to place proximals?
SVG may be anastomosed to innominate, carotid, SCA, R axillary artery, or descending aorta.
Be ready to circ arrest.

How well did you know this?
1
Not at all
2
3
4
5
Perfectly
83
Q

A CABG patient has porcelain aorta and severe AI. They need AVR. How do you make sure the heart doesn’t distend too much?
How do you do proximal SVG anastomoses?
What if you can’t find a safe place to clamp?

A

LV vent via R SPV or LV apex (also Axillary CPB. Retrograde arrest w/ eventual direct ostial vs graft plegia).
The patient will need ascending aortic replacement in order to place proximal anastomoses.
The patient may need circulatory arrest if you cannot find a safe place to clamp.

How well did you know this?
1
Not at all
2
3
4
5
Perfectly
84
Q

Redo CABG has higher risk than primary revascularization w/ mortality of 7-11% mostly 2/2 what disease?
What causes this?

A

perioperative MI - can be 2/2 incomplete revascularization, atheromatous emboli, damaged grafts, hypoperfusion through new grafts, or early graft occlusion

How well did you know this?
1
Not at all
2
3
4
5
Perfectly
85
Q

In a redo CABG, what is the general principle in regard to handling previous venous bypass grafts?

A

No touch technique - avoid embolization of debri

How well did you know this?
1
Not at all
2
3
4
5
Perfectly
86
Q

What is your myocardial protection strategy for redo CABG?

A

Antegrade alone may not protect areas supplied by patent pedicled IMAs and may dislodge debri in SVG.
Retrograde allows washout of coronary debris and access to myocardial areas of occluded arterial grafts.
Clamping of patent arterial grafts ensures uniform cooling. Do NOT do difficult dissection of open LIMA-LAD if risk injuring it.
Consider cooling pt to 28-30 if keeping LIMA patent.

How well did you know this?
1
Not at all
2
3
4
5
Perfectly
87
Q

During redo CABG, what strategy can be utilized in order to minimize the number of proximal anastomoses (ie the proximal sites on aorta are limited d/t prior grafts)?

A

sequencing vein grafts

How well did you know this?
1
Not at all
2
3
4
5
Perfectly
88
Q

During redo CABG, patient fails to wean from bypass. Doppler is done once all other causes ruled out, and poor flow is found with associated regional abnormality on TEE. What do you do?

A

Grafts to that area on TEE should be reconstructed immediately.

How well did you know this?
1
Not at all
2
3
4
5
Perfectly
89
Q

During redo CABG, there is no room on the aorta for proximal anastomoses. What options are there?

A

End-to-side to patent arterial grafts.
Use RIMA in situ if possible.

How well did you know this?
1
Not at all
2
3
4
5
Perfectly
90
Q

During redo CABG sternotomy, there is bright red blood encountered before sternum is completely open. There are EKG changes.
What do you expect?
What do you do?

A

Suspect coronary/graft injury (bright red, unlike dark blood in innominate injury).
Heparinize and obtain femoral bypass (there’s still too much dissecting to do to just attempt opening for central bypass).
Can attempt repair once injury is exposed vs coronary perfusion catheter.
Complete the operation.

How well did you know this?
1
Not at all
2
3
4
5
Perfectly
91
Q

During REDO CABG, you are trying to dissect the LIMA-LAD proximally, and you injure it. There are hemodynamic, EKG, and wall motion abnormalities. How do you manage?

A

Get on bypass. Clamp and arrest the heart. Try to repair the injured mammary.
*Before dissection of the mammary, you need to be ready to clamp and arrest - heparinized w/ high ACT, cannulated, dissected ascending aorta.

How well did you know this?
1
Not at all
2
3
4
5
Perfectly
92
Q

What are the important diagnostic studies for post-infarct VSD?

A

EKG - rule out ongoing ischemia and arrhythmia.
Echo diagnostic - color flow Doppler shows size and location of VSD (also valves, R side pressures, PA, tamponade).
RHC - step-up in oxygenation b/w RA and PA is diagnostic >9%; can also see elevated pulm-to-systemic flow ratio (1.4:1 to 8:1).
LHC - can determine if need for revasc.

How well did you know this?
1
Not at all
2
3
4
5
Perfectly
93
Q

Post-infarct VSD mortality?

A

Poor - 25% within 24 hrs, increasing with time up to 97% in 1 year.
This is an indication for urgent surgery.

How well did you know this?
1
Not at all
2
3
4
5
Perfectly
94
Q

Manage a post-infarct VSD preop.
What are your hemodynamic goals and what meds or methods do you use to achieve them?

A

Reduce afterload to decrease L-to-R shunt (nitroprusside).
Maintain cardiac output and perfusion (milrinone).
Increase coronary perfusion pressure.

IABP and inotropes.
ECMO or biventricular support can be used for temporary salvage before more definitive surgery can be performed.

How well did you know this?
1
Not at all
2
3
4
5
Perfectly
95
Q

How do you do an anterior septal rupture repair (pt w/ post-infarct VSD)?

A

No PA catheter.
Conduit harvest if needed.
Bicaval cannulation. Antegrade cardioplegia. LV vent in R SPV.
Cool to 25 C.
LV transinfarct incision and infarctectomy including septum.
Patch with horizontal mattress sutures, felt strip, and Dacron patch.
Deair, wean CPB, TEE to assess residual VSD/shunt/LV fct/MR.
+/- IABP.

How well did you know this?
1
Not at all
2
3
4
5
Perfectly
96
Q

How do you do apical VSD repair (postinfarct VSD)?

A

Incision through infarcted apex.
Debride necrotic tissue (may include LV, RV, septum).
Reapproximate w/ interrupted mattress through felt (use felt b/w each layer).

How well did you know this?
1
Not at all
2
3
4
5
Perfectly
97
Q

How do you do posteroinferior VSD repair (postinfarct VSD)?

A

CPB and arrest.
The heart needs to be retracted (like for PDA bypass).
LV transinfarct incision w/ 1 cm of space lateral to PDA.
Debride necrotic tissue.
Inspect mitral for papillary infarct.
Inspect posterior septum - can re-approximate using double layer buttress.
Large defects require patch (must be tension free).

How well did you know this?
1
Not at all
2
3
4
5
Perfectly
98
Q

When is delayed repair for post-infarct VSD acceptable?

A

Severe end-organ damage unable to undergo operative repair.
May benefit to use Impella or IABP to improve end organ dysfunction and allow for maturation of the infarcted tissue.

Biventricular assist device may be needed if LVAD worsens R-to-L shunt.

Can also improve forward flow and decrease afterload on LV with Impella/IABP while using ECMO to decrease load on RV and oxygenate blood to bypass the VSD.

How well did you know this?
1
Not at all
2
3
4
5
Perfectly
99
Q

What is the greatest predictor of post-operative mortality in pts w/ postinfarct VSD?

A

Time in cardiogenic shock

How well did you know this?
1
Not at all
2
3
4
5
Perfectly
100
Q

Is asymptomatic severe aortic stenosis an indication for surgery?

A

Previously, not on its own.
New guidelines state that you can.

Absolute surgical indications: low EF, sx (with or without stress testing), velocity >4.5.

If intervention required, transfemoral TAVI should be considered in following settings: STS-PROM >8%, or 4-8 w/o high-risk anatomic features, or low risk w/ age >65; transfemoral feasible, trileaflet valve, absence of high-risk feaures.

How well did you know this?
1
Not at all
2
3
4
5
Perfectly
101
Q

How can you “unmask” asymptomatic severe aortic stenosis to objectively confirm a patient is without impairment?

A

exercise stress test (treadmill) - positive if…
- Symptoms.
- SBP dec by >10 mm Hg.
- Dysrhythmias.
- ST changes.

How well did you know this?
1
Not at all
2
3
4
5
Perfectly
102
Q

What is the peak gradient equation (in the context of aortic stenosis)?

How is it directly obtained/measured (as opposed to calculated as above)?

A

Peak gradient = 4(velocity)^2.

Maximum gradient present when central aortic pressure is subtracted from LV systolic pressure.
Obtained by echo w/ continuous wave doppler. Or during LHC.

How well did you know this?
1
Not at all
2
3
4
5
Perfectly
103
Q

In what scenario may a patient with severe aortic stenosis have a low gradient?

A

In a patient with low cardiac output. Velocity, and thus gradient (4[velocity]^2), is affected by CO. This is a low gradient/low flow AS.

How well did you know this?
1
Not at all
2
3
4
5
Perfectly
104
Q

How can a low flow/low gradient AS be confirmed in a patient w/ normal EF (>50)?

A

Stroke volume index <35% ml/min/m^2.

How well did you know this?
1
Not at all
2
3
4
5
Perfectly
105
Q

How can a low flow/low gradient AS be confirmed if the patient has a low EF (<50%)?

A

Dobutamine stress echo - if gradient increases while AVA remains the same, AS is confirmed.

How well did you know this?
1
Not at all
2
3
4
5
Perfectly
106
Q

If low flow/low gradient AS is suspected in a patient w/ low EF (<50%), and dobutamine stress echo shows NO change in gradient w/ AVA increase >0.3cm^2 (or if AVA reaches 1cm^2), what is the diagnosis?

A

pseudo-AS

How well did you know this?
1
Not at all
2
3
4
5
Perfectly
107
Q

In the workup of AS, a significant subvalvular gradient and asymmetric septal hypertrophy is found, what treatment may be required?

A

Septal myectomy and possible mitral valve surgery

How well did you know this?
1
Not at all
2
3
4
5
Perfectly
108
Q

If the diagnosis of AS is uncertain by echo, including dobutamine stress testing, what else can be done?

A

Cardiac cath to cross the valve and obtain a hemodynamic AV study.

Can directly measure gradients.

How well did you know this?
1
Not at all
2
3
4
5
Perfectly
109
Q

In a potential TAVR candidate for AS, what imaging needs to be obtained?

A

TAVR CT scan - valve morphology (TTE unreliable), AV annulus and LVOT sizing, coronary height and orientation, and sinus height and width sizing.

How well did you know this?
1
Not at all
2
3
4
5
Perfectly
110
Q

What are the diagnostic criteria for severe AS?
What else should you rule out?

A

Any of the following:
- mean AV gradient >40
- peak velocity across AV >4 m/s
- AVA <1 or indexed <0.6
- dimensionless index <0.25
Always confirm. Always r/o CAD, aneurysm, and MV disease.

How well did you know this?
1
Not at all
2
3
4
5
Perfectly
111
Q

How do you decide on SAVR vs TAVR for symptomatic, severe symptomatic AS?

A

Guidelines say you can choose either, but considerations are as follows:
Anatomy.
Age and medical conditions affecting long-term prognosis.
Preference.
It was previously trending that in principle, you should rule out TAVR as an option (anatomy etc) unless there are other indications for surgical intervention. The pendulum is swinging the other way.

Stage B (Vmax 3-3.9) w/ other indication gets SAVR.
Stage C (Vmax 4) w/ ETT showing dec BP or dec ex cap, or high severity (Vmax 5, BNP 3x nl, rapid progression) + low surgical risk gets SAVR.

How well did you know this?
1
Not at all
2
3
4
5
Perfectly
112
Q

What STS risk score is prohibitive for SAVR for symptomatic severe AS?

A

STS >15%. TAVR shows survival benefit in these patients.
>8% seems to favor TAVR as well.

How well did you know this?
1
Not at all
2
3
4
5
Perfectly
113
Q

Compare likely complications for TAVR vs SAVR.

A

TAVR - perivalvular leak, ppm.
SAVR - bleeding, re-hospitalization, atrial fibrillation.

How well did you know this?
1
Not at all
2
3
4
5
Perfectly
114
Q

What are the benefits of a mechanical aortic valve compared to bioprosthetic?

A

Lower reintervention rate in <60 yrs old.
Superior survival at 15 yrs if b/w 50-70 yrs.

How well did you know this?
1
Not at all
2
3
4
5
Perfectly
115
Q

What complication of a surgical bioprosthetic aortic valve placement will likely reduce benefit from a valve-in-valve procedure?

A

patient-prosthesis mismatch

How well did you know this?
1
Not at all
2
3
4
5
Perfectly
116
Q

An AS patient undergoes TAVR CT showing bicuspid aortic valve. The valve is functional, and the patient is low risk. Discuss TAVR use.

A

TAVR initially not approved for use in this population.
But new data is showing comparable outcomes of TAVR is similar to SAVR in BAV AS patients.
Surgery remains first-line for AS BAV patients, but TAVR with the latest prostheses may be a safe alternative in patients with increased risk for surgery after meticulous workup of AV anatomy.

How well did you know this?
1
Not at all
2
3
4
5
Perfectly
117
Q

A patient w/ AS and intermediate surgical risk has an associated ascending aortic aneurysm. At what size is there a strong indication for aorta surgery?

A

4.5 cm

How well did you know this?
1
Not at all
2
3
4
5
Perfectly
118
Q

What is the EOA and why is it useful for aortic valve placement?

A

Effective orifice area (obtained by sizing valve and finding the corresponding EOA published by the manufacturer)/BSA.
Used to predict PPM.

How well did you know this?
1
Not at all
2
3
4
5
Perfectly
119
Q

An aortic valve’s EOAI is < 0.85. What does this mean?

A

Predictive of PPM. 0.65-0.85 is predictive of moderate PPM. <0.65 is severe.

How well did you know this?
1
Not at all
2
3
4
5
Perfectly
120
Q

Is EOAI <0.85 a contraindication for aortic valve surgery?

A

No. In a comorbid elderly patient w/ symptomatic severe AS, a risk of moderate PPM is preferred over cross-clamp time and surgical risk (ie TAVR w/ small EOAI is ok).

Should try to do an AV annular enlargement if at all possible in a surgically fit patient.

How well did you know this?
1
Not at all
2
3
4
5
Perfectly
121
Q

During workup for CABG, moderate AS is found. How do you manage?

A

Class IIa indication for replacement at time of surgery.

How well did you know this?
1
Not at all
2
3
4
5
Perfectly
122
Q

You are doing surgery for AS, but the heart is not arresting well. How do you manage?
What do you have to rule out?

A

Make sure the heart isn’t blowing up.
Check crossclamp.
Check drainage.
May just be slow d/t hypertrophy.
Add LV vent in R SPV.
Open aorta and give direct cardioplegia.
Cool patient.
May need higher dose of cardioplegia.

How well did you know this?
1
Not at all
2
3
4
5
Perfectly
123
Q

Intraop TEE during AVR for AS shows moderate perivalvular leak. How do you manage?
What are you looking for?
If having to completely redo, what do you need to make sure of on reattempt?

A

Clamp, arrest, open aorta, and reassess.
If visible defect, place a stitch.
If not, remove valve and start over.
Ensure decalcification of entire annulus (retained Ca can lead to improper seating). May need to reconstruct with pericardium.

How well did you know this?
1
Not at all
2
3
4
5
Perfectly
124
Q

What can you do if a patient’s EOAI is <0.85 during AVR for AS, and this is unacceptable?

A

consider root enlargement or total root replacement

How well did you know this?
1
Not at all
2
3
4
5
Perfectly
125
Q

What can make an asymptomatic severe AS more of a SAVR candidate (higher grade of recommendation) as opposed to TAVR?

A

Low gradient w/ <50% EF.
Critical AS (>5m/s velocity).
Severe LVH.
Concomitant cardiac operation.
Severe calcification.

How well did you know this?
1
Not at all
2
3
4
5
Perfectly
126
Q

A patient with aortic regurgitation gets an echo. What are you looking for?

A

Confirm diagnosis and severity, assess for etiology, valve morphology, LV assessments, aortic root size.

How well did you know this?
1
Not at all
2
3
4
5
Perfectly
127
Q

In workup for AR, you find a bicuspid valve with prolapse leaflets and minimal calcification.
What does this tell you what about the expected procedure?
What is often an associated pathology?

A

More amenable to repair, especially with more normal sized roots and without calcification.
The root (ie associated w/ ascending aortic aneurysm and dissection) is often affected, but a valve-sparing root replacement with valve repair is very effective in this population.
Jet direction is critical.

Of 85 valve-sparing root replacements for bicuspid aortic valve aneurysm disease, 99 percent were free of more than moderate AR at eight year.

How well did you know this?
1
Not at all
2
3
4
5
Perfectly
128
Q

Which AR patients need a heart cath?

A

Need: significant risk factors for CAD or those older than 40 in whom AVR is considered.
It may be good to assess RV/pulm HTN as well.

How well did you know this?
1
Not at all
2
3
4
5
Perfectly
129
Q

In workup of AR, what test can be done to confirm diagnosis if echo is inconclusive or discordant w/ physical findings?

A

LHC w/ root angiography and measurement of LV pressures

How well did you know this?
1
Not at all
2
3
4
5
Perfectly
130
Q

Severe AR is defined by what echo/angio parameters?

A

jet width >65% of LVOT
vena contracta >0.6 cm
holodiastolic flow reversal in the prox abdominal aorta
regurgitant volume >60 ml/beat
regurgitant fraction >50%
effective regurgitant orifice >0.3 cm2
angio grade 3+ or 4+

How well did you know this?
1
Not at all
2
3
4
5
Perfectly
131
Q

Diagnosis of chronic severe AR requires evidence of what?

A

LV dilation.

Be more cautious in these patients. The heart is weakening, so be prepared to support (IABP, ECMO, Impella; pressors).

The R heart may also be weak, and there could be pulm HTN to deal with.

How well did you know this?
1
Not at all
2
3
4
5
Perfectly
132
Q

guideline indications for surgery for chronic AR?

A

Symptoms AND severe.

ASYMPTOMATIC, SEVERE, AND:
Other cardiac surgery (moderate AR is IIb).
LVEF <55 (no other cause identified).
Severe LV dilation - LVESD >50 mm (can’t squeeze) or indexed LVESD >25 (IIa).
LVEDD >65 (overfill) and nl EF (IIb).
Progressive decline in LVEF on 3 studies despite EF >55.

*of note, for chronic AR, there is no operating on asymptomatic nl EF patients w/o other evidence of dysfunction (LVESD, LVEFF, progression)
*all patients should get BP tx for goal systolic <140.
*nonop patients should get GDMT including ACEi/ARB.
*TAVI is not for SAVR candidates in AR (III: harm)

How well did you know this?
1
Not at all
2
3
4
5
Perfectly
133
Q

You are doing surgery in a patient w/ AR. CPB is initiated, and the heart continues to eject, and begins to distend. What do you do?

A

Prior to bypass, you should be ready for distention - have retrograde cardioplegia access.
Arrest immediately.
Create the aortotomy, then give direct cardioplegia.

How well did you know this?
1
Not at all
2
3
4
5
Perfectly
134
Q

When trying to establish retrograde cardioplegia for an AR case, you have difficulty, and the position is questionable. What can be done?

Assuming you’ve attempted using posterior palpation and TEE guidance.

A

Bicaval cannulation.
Make room for the cross clamp, and ID the aortotomy site.
Initiate CPB.
Place the LV vent, clamp, aortotomy, then give direct cardioplegia.
Once arrested, snare the SVC/IVC.

If the retrograde line is still needed, make a R atriotomy and place direct retrograde catheter for cardioplegia (just past opening to ensure middle vein gets plegia).

How well did you know this?
1
Not at all
2
3
4
5
Perfectly
135
Q

During AR case, what if you injure the coronary sinus and/or retrograde cardioplegia becomes unreliable?

A

Initiate CPB, clamp, give ostial cardioplegia.

How well did you know this?
1
Not at all
2
3
4
5
Perfectly
136
Q

During AR case, TEE and direct visualization shows a tricuspid valve w/ prolapse of the R cusp. What are the surgical options?

A

Replacement is safest.

How well did you know this?
1
Not at all
2
3
4
5
Perfectly
137
Q

During AV surgery for severe AR, a patient fibrillates and arrests while cannulating the RA. The heart dilates, and defibrillation fails. He is unable to be converted back to sinus. What do you do?

A

Institute CPB, empty the heart, and defibrillate again.
If unable to empty the heart d/t severe AI, cross clamp and open the aorta.
Decompress the heart with a sucker through the aortic valve and localize the coronary ostia to give antegrade cardioplegia w/ an ostial cannula.

How well did you know this?
1
Not at all
2
3
4
5
Perfectly
138
Q

You complete an AVR, but are unable to close the aortotomy d/t the large prosthesis putting tension on the aortotomy suture line. What do you do?

A

Use a bovine or autologous pericardial patch to ensure no tension on the suture line.

This can be avoided by not making the aortotomy incision too low and not trying to oversize the valve. Just do a root enlargement.

How well did you know this?
1
Not at all
2
3
4
5
Perfectly
139
Q

You complete an AVR, but there is periprosthetic regurgitation.
What do you do?
What does a leak around the non-coronary sinus imply?

A

Do a good diagnostic evaluation with TEE - quantify and localize the leak, differentiate between prosthetic vs periprosthetic.

You may have to remove the prosthetic to fix the problem.

Leaks from the non-coronary sinus are most amenable to direct suture repair.

How well did you know this?
1
Not at all
2
3
4
5
Perfectly
140
Q

Symptomatic acute aortic regurgitation should be taken care of expeditiously. Why?

A

The ventricle has not had time to develop any adaptation to overcome the increased volume which can lead to rapid deterioration.

How well did you know this?
1
Not at all
2
3
4
5
Perfectly
141
Q

In non-surgical candidates w/ AR being considered for TAVR, discuss the following:

Compared to AS disease… native valve characteristics in AR that would affect choice of TAVR self-expanding vs balloon expanding valve?

What is the mortality difference b/w TAVR for AR vs AS?

A

The lack of calcification and larger annulus size may mean a self-expanding prosthesis would be more successful.

The 30-day mortality for TAVR for AR is higher compared to TAVR for AS.

How well did you know this?
1
Not at all
2
3
4
5
Perfectly
142
Q

Describe the benefits of aortic valve repair in AR surgery?

A

Similar results for AVR w/o need for anticoagulation as opposed to mechanical implants and possibly improved durability compared to bioprosthetics

How well did you know this?
1
Not at all
2
3
4
5
Perfectly
143
Q

How do stented pericardial aortic valves tend to fail?

A

By aortic stenosis of the biologic prosthesis, but tend to last longer than porcine valve.

How well did you know this?
1
Not at all
2
3
4
5
Perfectly
144
Q

What type of aortic valve is generally favored in younger (<55) patients?

A

Mechanical valves, though they do limit lifestyle d/t need for anticoagulation, and they limit future TAVR options.

Ideally, <50 gets a Ross.

How well did you know this?
1
Not at all
2
3
4
5
Perfectly
145
Q

What tests should you obtain before a redo AVR?

A

Echo (consider TEE if concern for endocarditis).
Coronary imaging.
CTA C/A/P to eval for transcatheter options, root size, SJ width, coronary heights; access options.
Carotid duplex
PFTs if hx of pulm disease
Vein mapping if previous CABG
Prev op note: when, what was last valve, other procedures (root enlargement, ascending aorta), CABG conduits and if crossing midline

How well did you know this?
1
Not at all
2
3
4
5
Perfectly
146
Q

Compared to TAVR, redo AVR is associated with what risk profile?
Ie compare risks of specific complications.

A

Redo AVR has…
Lower vascular complications and perivalvular leak.
Higher short-term stroke rate, atrial fib rate, AKI, bleeding.

How well did you know this?
1
Not at all
2
3
4
5
Perfectly
147
Q

How do you decide b/w valve in valve TAVR vs redo SAVR?
Pros of ViV TAVR? Contraindications?

A

Early complication rates are lower for ViV TAVR. Durability is unknown.
Endocarditis is a contraindication.
Mechanical valves preclude TAVR.

Eval for risk of patient-prosthesis mismatch: What were the gradients postop? If they are high, the original valve may already be small => redo SAVR +/- root enlargement. If they are nonoperative, then the patient will have to tolerate some PPM.

Eval for risk of coronary obstruction: coronary height needs to be >10mm.

How well did you know this?
1
Not at all
2
3
4
5
Perfectly
148
Q

In considering valve-in-valve TAVR, what would be considered a small in place prosthetic valve?

A

<23 mm may require the smallest TAVR inside or require fracturing of the surgical valve via high pressure balloon dilation.
At <23 mm, there should be a compelling reason not to do redo SAVR.
Similar issues: narrow STJ, small root.

**when performing bio SAVR, try to get at least a 23 valve in as to not eliminate future ViV option. This may be a reason in itself to do a root enlargement, and even with an ok EOAI, a small valve does not allow for much exercise.

How well did you know this?
1
Not at all
2
3
4
5
Perfectly
149
Q

In terms of a valve-in-valve TAVR, what measurement/diameter must be known in terms of the TAVR placement?
How can this number be confirmed?

A

The internal diameter of the previous surgical valve.
Look up product guide to find the internal diameter, and confirm w/ CTA or TEE.

When evaluating for SAVR, the external diameter of the valve matters as it must fit in place of the native valve annulus.

How well did you know this?
1
Not at all
2
3
4
5
Perfectly
150
Q

A previous AVR fails d/t AI (as opposed to AS). What are the considerations for procedural management (TAVR vs SAVR)?

A

There needs to be a compelling reason to NOT do redo SAVR.
Most current TAVRs are designed for AS, not AI.

How well did you know this?
1
Not at all
2
3
4
5
Perfectly
151
Q

What cardiac surgery history and medical history should cause a stronger consideration for ViV TAVR as opposed to a redo AVR?
What about imaging findings?

A

Prior CABG w/ LIMA (esp if near sternum).
HFrEF, >75-80 yrs, poor condition, renal failure.
Calcified ascending aorta.

How well did you know this?
1
Not at all
2
3
4
5
Perfectly
152
Q

ViV TAVR self-expanding vs balloon expandable discussion.
Self-expanding pros (2).
Self-expanding con (1).
How is this con decreased in this specific setting?

A

Self-expanding:
Better gradients (important since valve area is being reduced more d/t ViV).

Less traumatic in setting of calcified aorta.
Do not require rapid pacing (important if HF/sick ventricle 2/2 recurrent AS).

Higher pacemaker risk (somewhat mitigated in ViV by previous valve “protecting” conduction system when compared to first time).

How well did you know this?
1
Not at all
2
3
4
5
Perfectly
153
Q

For a younger patient at low surgical risk w/ severe structural bioprosthetic aortic valve deterioration, what is the ideal procedure?
What aortic valve and aortic root characteristics would make this patient more likely to benefit from the above answer?

A

Redo AVR.

Especially if AR.
Especially if small valve (<23 mm), narrow STJ, or small root. Clue would be if initial postop gradients from previous SAVR are high.
Especially if high risk for coronary obstruction (eg surgical valve has leaflets outside the frame, effaced sinuses, low coronary height [<10 mm]).
Especially if other concomitant surgical pathology.

How well did you know this?
1
Not at all
2
3
4
5
Perfectly
154
Q

Discuss cannulation strategy for redo AVR.

A

Be prepared for anything - have preop imaging; axilla, groins, and legs prepped (vein if coronary injury).
Consider placing femoral sheaths (4-5 Fr).
Consider CPB prior to opening in following situations - aorta close to sternum, high PA pressures w/ RV close to sternum, critical bypass grafts under sternum, calcified aorta.

Otherwise, with careful dissection, can do central cannulation.

How well did you know this?
1
Not at all
2
3
4
5
Perfectly
155
Q

Discuss myocardial protection in a redo AVR w/ AI.

A

Can arrest initially w/ retrograde cardioplegia then open aorta and deliver the rest as ostial plegia to ensure good protection.
Can try arresting antegrade, but it’s difficult to estimate how much is truly going down coronaries vs going into LV.

How well did you know this?
1
Not at all
2
3
4
5
Perfectly
156
Q

What are the considerations for the conduction system in redo AVRs (inc risk for what, and should consider what to prevent complications)?

Where is the conduction system in relation to the aortic valve?

A

Higher risk of heart block. Make sure you have reliable wires, and consider a-wires.

Near the underside of the R/non commissure.

How well did you know this?
1
Not at all
2
3
4
5
Perfectly
157
Q

Discuss prevention and management of VF during initial dissection of a redo AVR w/ moderate AI.
What causes this?
What preventative measures can you take?
What if the patient continues to be unstable or the heart distends in the incompletely dissected heart?
What if the heart distends?

A

Can be caused by CPB w/ an incompetent valve or bovie near the LV during dissection (keep cautery <45).

Have external pads, so you can shock prior to heart being dissected out (100-200 joules).

Having heart dissected out before CPB allows you to shock w/ paddles in case of fib during CPB.

Instability - cannulate and go on pump immediately.

Heart distends - LV vent (ideally R SPV; LV apex if exposed; or clamp aorta and aortotomy w/ ostial plegia)

How well did you know this?
1
Not at all
2
3
4
5
Perfectly
158
Q

Discuss operative plan for redo AVR w/ previous CABG.

A

Need preop cardiac cath and study of previous op note.
LIMA patent - frequent plegia and cool to 30 vs clamp LIMA (must be easy dissection, don’t risk injury)
- can clamp across all tissue w/ vascular clamp,
- if dissecting LIMA, be ready to clamp and arrest because injury will require immediate CPB and plegia
May need to mobilize grafts for aortotomy vs transect and rebypass w/ ligation of the old graft.

How well did you know this?
1
Not at all
2
3
4
5
Perfectly
159
Q

During redo AVR, you are unable to access the root and get exposure of the valve via aortotomy, what can be done next for exposure?

A

Transect aorta.

How well did you know this?
1
Not at all
2
3
4
5
Perfectly
160
Q

During redo AVR, you tear the aorta/outflow tract when removing the old valve. How do you fix this?

A

Patch.
May need homograft recon if extensive.

How well did you know this?
1
Not at all
2
3
4
5
Perfectly
161
Q

During redo AVR, you injure R coronary ostia while removing the valve. How do you manage?

A

Preparation: have legs prepped for any root case so vein can be taken.

Look at cath to ensure you are distal to disease during bypass.

How well did you know this?
1
Not at all
2
3
4
5
Perfectly
162
Q

Coming off bypass from redo AVR, and there is RV dysfunction. Assessment and plan?

A

Always assume regional functional issues are coronary problems.
Rule out air in the coronary system.
Assume new valve obstructed R coronary ostia - go back on pump, and graft R coronary system.

DO NOT reopen aorta and redo valve. You should have made sure you could see coronary ostia before closing aortotomy.

How well did you know this?
1
Not at all
2
3
4
5
Perfectly
163
Q

How can you be sure coronary ostia are open after redo AVR?

A

Before closing aortotomy, visualize the ostia.
Give retrograde plegia and look for blood return from the os.

How well did you know this?
1
Not at all
2
3
4
5
Perfectly
164
Q

After redo AVR, you cannot close the aortotomy d/t tension on the aorta.

A

In redo settings, the aorta can be less compliant. Use a patch, and try not to “oversize” the aortic valve.

How well did you know this?
1
Not at all
2
3
4
5
Perfectly
165
Q

77yr pt w/ mechanical AVR has failed w/ AS to the point of requiring intervention, and there is MR. CKD. Assessment and plan?

A

Likely functional MR 2/2 AS.
The principle is that the culprit lesion is the AS. In this difficult patient, it’s best to think of the safest option…
Safest: redo SAVR and monitor MR as outpt for need for Mitra Clip.

How well did you know this?
1
Not at all
2
3
4
5
Perfectly
166
Q

What critical diagnoses need to be considered in the differential for patients with a failed bio AVR being considered for re-intervention (possible ViV TAVR vs SAVR considerations)?

A

PPM - look for evidence of high gradients postop => may need enlargement and SAVR over ViV TAVR
Endocarditis - echo and cultures in right clinical setting => no ViV TAVR for IE

How well did you know this?
1
Not at all
2
3
4
5
Perfectly
167
Q

In terms of valve sizing, what info can be obtained before AVR to ensure no PPM?

A

Check BSA and CTA. If the required size is unlikely to fit the patient’s anatomy, be prepared for a root enlargement.
EOA (obtained via manufacturer) of valve / BSA should be > 0.80 to prevent PPM.

How well did you know this?
1
Not at all
2
3
4
5
Perfectly
168
Q

Primary mitral regurgitation is generally caused by what?

A

primary leaflet abnormalities - usually result of degenerative disease

How well did you know this?
1
Not at all
2
3
4
5
Perfectly
169
Q

Secondary MR is generally caused by what?

A

structurally normal valve leaflets are unable to coapt completely - owing to a distortion of either the annulus or the subvalvular apparatus

can be from ischemic cardiomyopathy or other causes of dilated cardiomyopathy

How well did you know this?
1
Not at all
2
3
4
5
Perfectly
170
Q

LV dysfunction and secondary MR prognosis compared to either abnormality in isolation?
Is revascularization or addressing MR of value?

A

much worse; controversial whether revascularization alone is helpful; controversial whether addressing MR will be of value, but data seems to suggest that it does help

How well did you know this?
1
Not at all
2
3
4
5
Perfectly
171
Q

Carpentier’s Type 1 MR?

A

normal leaflet motion - results from LV enlargement and annular dilation (functional or secondary MR)
can also be from perforation in leaflet

How well did you know this?
1
Not at all
2
3
4
5
Perfectly
172
Q

Carpentier’s Type II MR?

A

leaflet prolapse - can be degenerative, acute papillary rupture after MI

How well did you know this?
1
Not at all
2
3
4
5
Perfectly
173
Q

Carpentier’s Type IIIa MR?

A

leaflet restriction, systole and diastole - rheumatic

How well did you know this?
1
Not at all
2
3
4
5
Perfectly
174
Q

Carpentier’s Type IIIb MR?

A

restriction during systole alone - chronic ischemic disease

How well did you know this?
1
Not at all
2
3
4
5
Perfectly
175
Q

Dilated cardiomyopathy is likely to cause what type of MR?

A

type 1 - normal leaflet motion
pts may have severely depressed LV fct

How well did you know this?
1
Not at all
2
3
4
5
Perfectly
176
Q

Ischemic MR tends to cause what type of MR?
What is mechanism?

A

Type IIIb - systolic restriction of leaflets.
Chronic ischemia can cause asymmetric ventricular remodeling => mostly affects inferior and lateral LV wall => disruption of the subvalvular apparatus => downward and lateral (apical) displacement of the PM papillary muscle => leaflet tethering.

How well did you know this?
1
Not at all
2
3
4
5
Perfectly
177
Q

What is the study of choice to clarify the mechanism of MR?
What is a problem with this study?

A

TEE - however, MR may be lower intraoperatively d/t anes and ventricular unloading

How well did you know this?
1
Not at all
2
3
4
5
Perfectly
178
Q

At the time of CABG for severe ischemic MR, what is the preferred surgery for the mitral valve?

A

mitral valve REPLACEMENT, chordal sparing - bio valve is probably best since poor prognosis long-term
*remember GDMT, as this is secondary MR
*stage D - RVol >60ml, RF >50%, ERO >0.4 cm2
*2a evidence
*either way, 2 yr survival is 20-25% - why bio valve is likely best

How well did you know this?
1
Not at all
2
3
4
5
Perfectly
179
Q

What is an Alfieri stitch and when is it appropriate?

A

At the time of apical incision such as LVAD for Type I (nl leaflet motion) secondary MR

How well did you know this?
1
Not at all
2
3
4
5
Perfectly
180
Q

In CABG MVR, why do you do distal anastomoses first before valve?

A

avoids excess manipulation of heart that predisposes to AV groove dissociation

How well did you know this?
1
Not at all
2
3
4
5
Perfectly
181
Q

An old pt w/ diffuse CAD and poor targets has dilated cardiomyopathy w/ secondary severe MR and EF of 15%. What should you plan for?

A

LVAD. Consider Alfieri suture after apical coring.

How well did you know this?
1
Not at all
2
3
4
5
Perfectly
182
Q

A patient who requires CABG is found to have moderate ischemic MR. What do you do for the MR?

A

Nothing. Concomitant MVr resulted in longer OR time, longer hospitalization, higher rates of a-fib, higher rates of stroke.
MitraClip may be better option down the road.

How well did you know this?
1
Not at all
2
3
4
5
Perfectly
183
Q

Pt w/ severe symptomatic secondary mitral regurgitation and EF <50%. Pt sx are persistent despite GDMT. What is recommended for mitral valve?

If surgery, discuss repair vs replacement.

A

Eval for transcatheter edge-to-edge MV repair (2a)
*LVEF 20-50
*LVESD <70mm
*PASP <70 mmHg

If anatomy not good, then consider surgery (2b)

In an RCT of mitral valve repair versus mitral valve replacement in patients with severe ischemic MR, there was no difference between repair and mitral valve replacement in survival rate or LV remodeling at 2 years. However, the rate of recurrence of moderate or severe MR over 2 years was higher in the repair group than in the replacement group, leading to a higher incidence of HF and repeat hospitalization. The lack of apparent benefit of valve repair over valve replacement in secondary MR versus primary MR, with less durable repairs in secondary MR, highlights that primary and secondary MR are 2 different diseases. (2020 ACC/AHA).

How well did you know this?
1
Not at all
2
3
4
5
Perfectly
184
Q

What is often the mechanism for acute mitral regurgitation 2-7 days after MI?
How do you manage?

A

Single blood supply to posteromedial papillary muscle - either RCA or L Cx depending on dominance.
Anterolateral has dual blood supply (LAD and L Cx).

Acute inferior STEMI -> Papillary muscle rupture -> flail leaflet -> apical holosystolic murmur, pulmonary edema -> dyspnea.

DO NOT increase afterload or cause volume overload. Give milrinone for inotropy and nitroprusside for vasodilation.
IABP should be considered.
ECMO may be needed.

Surgery: mitral replacement w/ chordal preservation.

*Can remember commissures by their blood supply: posteromedial has posterior (RCA) supply. Anterolateral has anterior (LAD) and lateral (LCx) supply.

How well did you know this?
1
Not at all
2
3
4
5
Perfectly
185
Q

How does the jet in primary mitral regurgitation predict leaflet affected?

A

anterior directed jet is from posterior leaflet prolapse and vice versa; when planning repair, need to know if pt can tolerate second pump run if repair fails

How well did you know this?
1
Not at all
2
3
4
5
Perfectly
186
Q

Vena contracta for severe mitral regurgitation?

A

0.7 cm

How well did you know this?
1
Not at all
2
3
4
5
Perfectly
187
Q

Regurgitant volume for severe MR?

A

60 ml

How well did you know this?
1
Not at all
2
3
4
5
Perfectly
188
Q

Regurgitant fraction for severe MR?

A

50%

How well did you know this?
1
Not at all
2
3
4
5
Perfectly
189
Q

Central jet (percentage of LA or characteristics) for severe MR?

A

40% LA or holosystolic eccentric jet

How well did you know this?
1
Not at all
2
3
4
5
Perfectly
190
Q

In primary mitral regurgitation, which patients should get an operation? Do they have to have severe findings?

A

This is for PRIMARY MR.
There are two conditions.
They have to be SEVERE AND have either…
- symptoms - this includes EF <30% IF REPAIR can be done.
- systolic dysfx: EF <60 (but >30), ESD >40.
- new PAH (PA MAP >20, PVR >3 Wood units) or new AF.
- inc LV size or dec in LV fct x3 studies.

OR a center of excellence. Surgery is favored if LA >50mm and/or PAH mild (30-50 systolic).
***These parameters should be adjusted if REPAIR cannot be done.

How well did you know this?
1
Not at all
2
3
4
5
Perfectly
191
Q

If doing a mitral valve repair, what is a necessary adjunct to reinforce most repair techniques?

A

Ring or band annuloplasty

How well did you know this?
1
Not at all
2
3
4
5
Perfectly
192
Q

During mitral repair, what is at risk when placing stitches near posteromedial commissure?

A

Near right trigone - conduction tissue

193
Q

During mitral repair, what is at risk if you place deep stitches at the anterior leaflet while moving from the right to left trigone?

A

Aortic valve (L/non commissure) - look for aortic insufficiency that wasn’t there preop.

194
Q

During mitral repair, what is at risk if you place deep sutures at P1?

A

Circumflex artery - look for WMA on the lateral wall.
Avoid by angling the needle toward the ventricle (away from the atrium).

195
Q

While doing a mitral repair, what is at risk if placing deep retraction stitches outside the annulus of the P3 scallop?

A

Coronary sinus

196
Q

What is likely the ideal repair in patients w/ mitral regurg 2/2 mitral valve prolapse 2/2 fibroelastic deficiency?

A

In fibroelastic deficiency, there is a paucity of tissue. Neochords are often the best repair, or a very limited triangular resection.

197
Q

In MVR w/ Barlow’s valve, what is often the best repair?

A

There is excess tissue in almost all scallops.
Perform quadrangular resection w/ sliding plasty.
(B) in picture***

198
Q

A pt with PRIMARY severe MR (vena contracta >0.7 cm, regurg volume >60, regurg fraction >50%, ERO >40%), presents to your clinic. She is asx w/ normal EF (>60%).
When does she get surgery?
What do you tell the surgeon who wants to do a replacement for primary MR?
What are options for secondary MR?

A

Any of the following:
- LV ESD >40.
- Mortality <1% and >95% of successful and durable repair.
- Progressive increase in LV size or decrease in LVEF >3 studies.
- If she develops sx or EF <60.
***DO NOT do replacement if leaflet pathology < 1/2 posterior leaflet (3: harm).

If severely symptomatic, and not surgical candidate, TEER is reasonable (life expectancy >1 yr, anatomy reasonable).

*2020 AHA

**If this was secondary MR, she would not be offered anything.

TEER could be offered for:
- Severe w/ EF <50 and failed GDMT (EF >20, LVESD <70, PASP <70). 2a.

Surgery for severe secondary MR:
- With CABG. 2a.
- MR 2/2 atrial dilation w/ EF >50 and persistent sx. 2b.
- EF <50 w/ persistent sx. 2b.

Chordal-sparing mitral valve replacement in CAD w/ severe secondary MR w/ EF <50.

199
Q

56F w/ hx of systolic heart murmur presents to ED w/ worse dyspnea and hemoptysis. She was doing intense physical work when she experienced severe precordial chest pain.
She is tachycardic w/ elevated JVP. She has a pansystolic murmur.
CXR shows BL pleural effusions.
What happened?
What are risk factors?
What is tx?

A

Exercise induced acute chordal rupture resulting in severe MR w/ heart failure.

The hx of murmur suggests an underlying structural problem - MVP puts pts at inc risk for rupture.
Other predisposing factors: rheumatic heart disease and infectious disease.
Blunt chest trauma is a rare cause.

OR for urgent mitral valve repair.

We know this is likely mitral d/t the acute overload presentation in the setting of a systolic murmur. AS is the only other systolic murmur like this, and acute native AS isn’t really a thing.

200
Q

In acute MR, acute chordal rupture from degenerative etiology needs to be differentiated from papillary muscle rupture d/t ischemia. Why?

A

The treatment is different.
Papillary muscle rupture often warrants replacement (they may also need a CABG).
Ruptured chords can be repaired.

201
Q

59M admitted w/ very acute and severe SoB w/ systolic murmur. CXR shows effusions BL w/ R lung opacification and leukocytosis. EKG and troponin normal.
What study should be done next?
What do you think it will show?

A

Get echo - can show severe MR 2/2 flail leaflet (AS would be less acute).
Do NOT misdiagnose PNA and start abx. The murmur means you need to determine if this is MR, or pt will decompensate.

202
Q

For MV approach, when might L atriotomy be best approach?
How is it done?

A

Most common approach.
Incision of choice for robotic and mini-thoracotomy.
Approach via R side through Sondergaard’s groove (Waterston’s).

203
Q

For MV approach, when might vertical transseptal be best approach?
How is this done?
Any advantages?
Any disadvantages?

A

Feasible w/ large LA in setting of chronic MR.
Via fossa ovalis.
Reduces risk of PPM, easier to close.
Downside is there are two suture lines, and you have to look for ASD on TEE after surgery.

204
Q

For MV approach, when might extended superior transseptal (Guiraudon) be best approach?
How is this done?
Disadvantages?
Any advantages?

A

Situation is when you’re in a redo case and can’t access Sondergaard. You attempt a vertical transseptal, but there isn’t enough exposure because LA isn’t large. Vertical transseptal incision is extended superiorly onto the dome of the LA through the limbus of the fossa ovalis.
Risk of damaging SA nodal artery (increased risk of PPM).
May provide better exposure for a deep chest anatomy/wide AP diameter.

Approach of choice in the setting of previous AVR or aortic root placement.
Can be easily extended and connected with an aortotomy incision for a Commando operation (endocarditis and radiation).

205
Q

For MV approach, when might horizontal transseptal (Dubost) be best approach?
Any advantages/disadvantages?

A

Uncommon.
Can be considered in the setting of a hostile anatomy b/w the aortic root and LA dome (presence of calcified aortic homograft).
Difficult closure, risk of narrowing R superior pulmonary vein, risk of transseptal incision progressing/tearing AV node -> high risk of PPM.

206
Q

For MV approach, when might transaortic be best approach?
How is this done?
Any advantages?

A

Aortotomy is extended onto the dome of the LA.
Useful in in endocarditis when both aortic and mitral valves need to be replaced (transaortic Commando).
Exposure can be challenging, in which case the incision can be connected to the extended transseptal incision.

207
Q

For MV approach, when might left atrial appendage be best approach?
How is this done?
Any advantages?

A

Very uncommon.
Left thoracotomy.
Alternative approach after multiple previous re-operations via R side.

208
Q

For MV approach, when might transventricular be best approach?
When is this done?

A

If entering the LV for another reason (ventricular aneurysmectomy or LVAD).

209
Q

Your patient is being evaluated for a mitral repair. They have a previous AVR complicated by a postop bleed requiring takeback then mediastinitis and sternal wound infection eventually requiring flap closure. How would you approach the mitral?
How do you get on bypass?
How do you arrest?

A

R anterolateral thoracotomy via 4th intercostal space centered at the anterior axillary line.
Cannulate through femorals.
Fibrillatory arrest under moderate hypothermia (requires competent AV).
Endoballoon (requires competent AV).

210
Q

A patient s/p mitral repair surgery is coming off pump. The patient becomes hypotensive, and 3+ MR is noted with encroachment of the anterior leaflet onto the LVOT.
What is the diagnosis and management?
What would the effect of nipride be in this setting (eg before coming off pump, anesthesia tries to “make room for volume”)?
What else could make this worse?

A

The patient has SAM.
Wait for volume and consider adding beta-blocker to ensure not simply underfilled.
Then aim to reduce the POSTERIOR leaflet height - folding valvuloplasty, revise triangular repair to quadrangular w/ sliding annuloplasty.
Place a larger ring.
If all fails, replace the valve.
If the patient is too frail to replace, do an Alfieri stitch.

Nipride would accentuate the problem by underfilling the ventricle thus increasing ventricular systolic contraction.
Inotropic support and reduced afterload could make things worse.

211
Q

Describe the pathophysiology of SAM after mitral surgery?

A

Mitral surgery changes the line of coaptation to displace it more anteriorly towards the LVOT.
The Venturi effect causes the anterior leaflet to get sucked into the LVOT during systole.
At the same time, the posterior leaflet prolapses towards the atrium, creating an anteriorly directed jet.
SAM thus causes both mitral regurgitation and LVOT gradient.

212
Q

What increases the risk of SAM in mitral surgery?
- posterior leaflet height
- hyperdynamic ventricle
- anterior leaflet relative size
- aorto mitral plane angle
- coapt to septum distance
- IVS thickness
- ring characteristics

A

Posterior leaflet is >1.5 cm in height after repair (ie it is tall).
Small hyperdynamic ventricle.
Ring is UNDERsized too much - causes line of coaptation to bulge anteriorly.
AL:PL ratio <1.3
Aorto-mitral plane angle <120 degrees (pic - the closer to 90 degrees, the more the anterior leaflet is pulled into LVOT).
Coapt/septum distance <25 mm
IVS thickness >15 mm
Anterior displacement of prosthetic ring.

213
Q

After MV replacement, bright red blood is welling up behind the heart after the cross clamp is released?
What is the likely diagnosis?
What is the cause?
What is the management?

A

AV dissociation after mitral valve replacement/ AV groove disruption.
Can happens up to hours after the procedure and is signaled by massive intrapericardial hemorrhage.
Rupture occurs in the LV near the AV groove posteriorly.
Tends to occur in women w/ small LVs.
The cause is technical:
- too much traction on the annulus during excision of the valve or insertion of the prosthesis
- lifting the heart after valve replacement, tearing the annulus
- stitching through the posterior AV groove
- perforation from papillary muscle excision
- perforation of the AV groove during calcium debridement
Management: go back on pump and re-arrest. Open the LA. Remove the prosthesis. Pericardial patch to recreate the annulus. Reimplant the valve in the newly created rim.
- if caused during repair, replace the valve and use the PL to buttress the patch repair and reduce tension on the suture

214
Q

Coming off pump from mitral surgery, ST changes are noted along the lateral leads (V5-V6) and the patient becomes hypotensive.
What happened?
What is the management?

A

Differential includes postcardiotomy syndrome and cardiogenic shock from reperfusion or poor protection.
Most likely is damage to the circumflex.
Management: back on pump, harvest vein, bypass to distal OM.

215
Q

During mitral surgery, the left atrium is opened, and there is blood. You cannot see anything.
What is your checklist?

A

Make sure there is a working vent/floppy sucker in the *RSPV.
Check the *aortic cross clamp - ensure it is all the way across and occluding flow.
Explore the septum for an *ASD - can be evaluated from LA, but may require *RA incision to repair.

216
Q

A patient has an indication for re-operative mitral valve replacement. What workup should be considered?

A

EKG - rule out AF
CXR - count sternal wires
CT - assess distance b/w sternum and RV/aorta/innominate vein, aortic Ca
CTA gated - if hx CABG, know course and distance of vessels
CT abd/pelvis - assess femorals and CPB course
Preop TEE - mitral pathology (especially for re-repair)
Cath - figure out CAD
RHC - if LV or RV dysfunction
Carotid duplex - older pts, vascular disease, coronary disease, neuro sx
PFTs - if COPD
***Op note - what was done to valve, valve manufacturer/size/material if replacement prev, how was it exposed

217
Q

Can a mitral valve be re-repaired?

A

Yes.
Re-repair is associated with lower mortality compared to replacement at reoperation.
Severe calcification (leaflets, annulus, subvalvular apparatus) and extensive leaflet destruction (e.g., endocarditis) can prevent re-repairability.
You will not be faulted for replacement in redo setting.

218
Q

How close is the circumflex to the PL mitral annulus?
When would you suspect injury?

A

2-4 mm.
Suspect injury: high-dose pressors when weaning. Pressor requirement persists. TTE shows lateral wall motion abnormality. EKG correlates w/ V5-V6 or 1 and AVL.

*
Right leads, V1-V2: Interventricular septum and right ventricle.
Anterior leads, V3-V4: Anterior wall of the Left ventricle.
Low lateral leads V5-V6: Low lateral wall.
High lateral leads I and AVL: High lateral wall.
Inferior leads II, III and AVF: Inferior wall.

219
Q

Redo mitral surgery is unable to be weaned from CPB w/o V-pacing. Monitor shows complete dissociation between atria and ventricles.
What is diagnosis? What happened?

A

Complete heart block 2/2 AV node damage 2/2 deep/wide posteromedial mitral commissure stitch during MVR or ring annuloplasty.

220
Q

What are the differences between early and late prosthetic valve endocarditis in terms of timing, causative bacteria, and mortality?

A

Early: within 2 mo, Staph, higher mortality
Late: Strep, lower mortality

221
Q

Discuss the difference in pathophys/anatomy of bio vs mechanical valve infection causing insufficiency.

A

Mechanical valve: infection localized to sewing ring -> abscess and dehiscence.
Bio: infection in leaflets -> vegetations, leaflet perforation

222
Q

A pt is undergoing redo bio MVR. She asks if she will need a 3rd reoperation. What do you say?

A

Freedom from reoperation at 15 yrs is 80% w/ bio valves in mitral position.

223
Q

What are the hemodynamic differences in bio mitral vs aortic valves in terms of their degeneration?
Are there other considerations in bio valve choice for young patients?

A

Mitral valves are exposed to increased hemodynamic stress during systole, so deteriorate faster.

Valve design has improved, and transcatheter valve-in-valve is an option to avoid re-operation.

224
Q

During MV replacement, what is the ideal management of the chords and leaflets?

A

Try to spare them - improved outcomes. But do not compromise outflow.

225
Q

Define severe symptomatic mitral stenosis.
What are measurements/echo findings?
What are associated symptoms?

A

Mitral valve area ≤1.5 cm2
Diastolic pressure half-time ≥150 ms
Severe LA enlargement
Elevated PASP >50 mm Hg

Decreased exercise tolerance
Exertional dyspnea

226
Q

How do you manage the anticoagulation in a pregnant woman with a mechanical valve?
How is the VKA risk profile?
What if anything else can or should be used?

A

Maintain therapeutic anticoag w/ frequent monitoring.
VKA has lowest maternal complications but highest likelihood of miscarriage, fetal death, and congenital abnormalities (esp 1st trimester and at dose >5).

Use LMWH instead to target anti-Xa 0.8 - 1.2.
Switch to UFH 6 hrs before delivery.
If urgent delivery needed, reverse VKA and do C-section.
(3:Harm - Must use anti-Xa for LMWH monitoring, dabigatran cannot be used, DOACs have not been assessed and shouldn’t be used).

227
Q

A pregnant woman with mechanical prosthetic valves undergoes an embolic event. What workup should they undergo?

How should they have been counseled pre-pregnancy?

A

TEE. Same goes if there is a prosthetic valve obstruction.

Pre-pregnancy: Should have gotten echo, seen cardiologist w/ expertise in pregnant VHD patients.
Mechanical VHD pregnant pts in particular should be at a tertiary-care center w/ MDT.
They should be told that pregnancy is high-risk, and that there is no anticoagulation strategy consistently safe for the mother and baby.

Women should receive therapeutic anticoag w/ frequent monitoring during pregnancy. If they cannot maintain therapeutic levels, they should be counseled against pregnancy.

228
Q

A pregnant women w/ severe valve regurgitation has NYHA class III or IV HF symptoms (stage D) refractory to medical therapy. What can be offered?

What they don’t have HF symptoms?

A

valve surgery (2a)

In pregnant pts w/ severe disease w/o severe and refractory HF symptoms (at rest or with min activity), valve surgeries should NOT be performed (3:harm).

229
Q

A pregnant female with severe AS has NYHA class III or IV symptoms. What should be done?

A

valve intervention is reasonable

AS = mean gradient 40, V max 4, AVA <1

230
Q

A pregnant female presents with new symptoms from known severe mitral stenosis. What should you do?

What should they have had done before pregnancy?

Do women who are considering pregnancy have different indications for intervention?

A

They should get TTE, be seen at a center w/ expertise, and consider exercise testing if asymptomatic and suspicious.
Meds to consider are beta-blockers, diuretics. DO NOT give ACEi or ARBs.
Intervention has standard indications. Prefer bio valves.

Best management would have been to counsel a pt of childbearing age that they should have their MS treated first.

MS = valve area <1.5 or diastolic PHT >150 (speed at which the LV fills in diastole based on the slope of the change in gradient)
1st: medical therapy.
2nd: PMBV.

231
Q

A pt w/ rheumatic mitral stenosis has severe (<1.5 cm) findings but is asymptomatic.
What workup do they need?
What are you looking for?
What are you trying to rule out with above workup?
What should they have done?

A

Figure out PASP and if AF.
So get TTE, TEE, LHC/RHC.
If PASP >50 mm Hg or New AF ->
Ensure no clot, valve pliable and not much MR (<2+) ->
PMBC - perc mitral balloon commissurotomy

232
Q

A pt w/ rheumatic mitral stenosis has severe (<1.5 cm) findings. What are the best options for treatment?

A

Commissurotomy - either percutaneous or open.

233
Q

What is the population of patients with rheumatic MS that should get SURGICAL commissurotomy?

A

Severe (<1.5 cm MVA) +
Symptomatic (dyspnea on exertion/dec exercise tolerance) +
Not candidate for PMBC (valve not pliable, clot, >2 MR) +
Surgical candidate.

234
Q

Which rheumatic MS patients should get intervention if they are asymptomatic.

A

Severe findings (<1.5 cm MVA) +
PMBC candidate (pliable valve, no clot, <2+ MR) +
New onset AF OR PASP >50 mm Hg.

235
Q

Is someone with progressive MS (>1.5 cm MVA) a candidate for MV intervention?
What are their options?

A

Yes. IF…
Exertional symptoms.
Confirmed: Stress test shows hemodynamically significant MS.
PMBC candidate: pliable valve, no clot, <2+ MR.

ONLY PMBC offered.

236
Q

A patient has severe MS and has severe symptoms. The junior fellow wants to do surgery. What should you tell them?

A

There needs to be a reason not to do PMBC.
Figure out if… valve is not pliable, clot is present, or MR is >2+.
Then they have to be surgical candidate.

237
Q

A patient with severe symptomatic mitral stenosis also has AS, TVR, and CAD that requires surgery.
What is the conduct of this operation?

A

Distal coronaries.
Aortic valve is excised - break it down.
MVR.
TVR.
AVR.
Proximal coronaries.

238
Q

How do you manage mitral annular calcification?

A

Usually old, frail, debilitated patients.
DELAY intervention until severely limiting symptoms unable to be managed w/ diuresis and HR control.
Some catheter-based strategies are being developed.
Surgery may require radical debridement of the annulus down to pericardial fat, reconstruction with pericardial patch that saddles/sandwiches the annulus, then valve placement with sutures from LV to patch to LA to sewing ring.

239
Q

A mitral valve surgery is completed, and the cross clamp is removed. The heart is beating, but the LV distends. What should be done to evaluate? What is the likely diagnosis? What should be done if this diagnosis is confirmed?

A

TEE. Aortic insufficiency 2/2 anterior annular mitral stitches in the non-coronary or L cusp.
Rearrest, open the aorta and LEFT atrium, and remove suture or redo the mitral surgery.
Repair the aortic cusp or replace it.

240
Q

How can you cause conduction block during mitral surgery?

A

Sutures too deep near the posterior leaflet near the PM commissure - bundle of His.

241
Q

After mitral surgery, what would cause increased gradients across the LVOT?
What do you do?

A

Prosthesis problem: high-profile mechanical valve, large stented bio valve posts, poorly seated valves.
Could also be SAM.
Either way… Diagnose on TEE, and may need to replace the valve w/ lower profile.

242
Q

What is a risk in mitral surgery if you leave the anterior leaflet unresected in a patient with septal hypertrophy?
What do you do?

A

SAM.
Stop inotropes, load, add betablocker, and add vasopressors - inc afterload and preload while reducing contractility (you can attempt these maneuvers for SAM after mitral surgery, but not after a myectomy for HOCM).
You may need to do an aortotomy and do a transaortic excision of the offending subaortic mitral tissue +/- myectomy.

243
Q

After mitral replacement, there is perivalvular leak on postop echo. How do you manage?

A

Small leaks stop after protamine.
Significant leaks require reimplantation or repair.

244
Q

What are important, specific postoperative hemodynamic and clinical parameters after MVR for MS?

A

Respiratory status - Many patients require aggressive diuresis.
Pulmonary pressures - Inotropy and pulmonary vasodilation may be required.

“High and dry.”

245
Q

What is the INR goal for mechanical MVR?

A

2.5-3.5.

This is higher than AC goal for atrial fibrillation, so it still increases risk in patients already on AC.

246
Q

What complication will occur in mitral surgery if there is aggressive debridement or retraction/lifting of the heart?
How do you prevent this?

A

AV groove disruption/rupture.

Prevent by minimizing retraction, preserving the posterior leaflet and its subvalvular apparatus, and using increased caution during debridement in pts w/ annular calcification.

247
Q

What are the three principal goals of surgical intervention for AF?

A

1) conduction block of all micro and macro re-entrant circuits
2) reestablishment or maintenance of electrical AV synchrony
3) restoration of atrial mechanical function in order to improve diastolic filling
(no re-entry circuits, good AV synchrony, good atrial kick)

248
Q

When can stand alone surgery be done for AF?

A

Symptomatic AF.
Refractory to class I/III AADs and/or catheter-based therapy (class IIA, level of evidence [LOE] B-R).

According to The Society of Thoracic Surgeons (STS) 2017 Clinical Practice Guidelines for the Surgical Treatment of Atrial Fibrillation, primary stand-alone SA is a reasonable procedure.

249
Q

During Cox Maze, you are dissecting the transverse sinus when you see dark blood beginning to pool. What is the likely injury?

A

R PA.

250
Q

A patient with ascending aorta aneurysm is undergoing aortic valve or CABG surgery. What is cutoff to add aorta intervention?

A

4.5 cm end-diastolic diameter.

251
Q

A pt w/ mitral regurg and atrial fibrillation undergoes mitral repair and Cox Maze IV w/ LAA. 2 wks postop, she has palpitations w/ a-flutter in 160s. What is happening?

A

Peri-mitral flutter.
Failure to interrupt conduction across the “LA isthmus” allows for this. It is the most common mode of failure of surgery for AF.

LA isthmus - linear lesion connecting the box to the mitral annulus.

252
Q

A pt has peri-mitral flutter after mitral repair w/ CMIV and LAA ligation. What is the cause and what can be done?

A

Conduction across the LA isthmus d/t incomplete lesion line involving the MV annulus.
EP can do catheter intervention to complete the ablation line.

253
Q

For AF, a large macro-reentrant circuit develops just above the mitral valve annulus. Which lesions must be completed in regards to preventing ongoing flutter (peri-mitral flutter)?

A

Mitral line (prevent conduction across the atrial myocardium).
Coronary sinus lesion (prevent conduction across the coronary sinus; collapse the CS during cryoablation so a circumferential lesion can be assured; should connect to the level of the mitral annulus).

Place both lesions in the same plane. Transmural ablation (epicardium and endocardium) is key.

254
Q

While performing CMIV, you notice a cryolesion across the pericardial surface of the RA. What complication can you expect?

A

Hemidiaphragm paralysis from damage to the R phrenic. This is usually temporary.
Make sure the handle of the probe is not in contact with the phrenics (R or L).

255
Q

What are the principles of ensuring a CMIV doesn’t fail?

A

A single break in the line can preserve a macro reentry circuit. These waves will find the weakness.
Lesions should be uniformly transmural and contiguous.

Ensure minimal tissue folding.
Clean the tips of the clamp.
Multiple burns per line.

256
Q

In CMIV surgery, can char on the RFA clamp affect outcome?

A

Yes. The RF clamp relies on perfect contact b/w the tissue and electrodes. Char isolates and disrupts this contact, which can affect tissue from heating, breaking the line, and allowing macro-reentrant circuits through.
It is recommended to clean the clamp after every 3 ablations.

257
Q

What is the ligament of marshall?
What is its importance in AF/CMIV?

A

It’s an epicardial vestigial fold that marks the location of the embryologic LEFT SVC. It contains the nerve, vein (vein of Marshall), and muscle tracts.
It is located on the epicardium between the left atrial appendage and the left pulmonary veins. The corresponding endocardial structure is the LA ridge.

It is a source of paroxysmal AF.

It helps to dissect it out to get better atrial clip during CMIV.
Watch out for L Cx.

258
Q

In CMIV procedure, you cannulate, place the pt on bypass, and place an LV vent. You are exposing the L heart for LPV isolation. You divide the ligament of Marshall, then notice a device protruding through the LAA. What happened?

A

Rare. You may have compromised the position of the LV vent by accidentally entering the LA.

It is still necessary to divide the LoM to access behind the L pulmonary antrum for L PV isolation. Close the defect. Finish the case.

259
Q

You perform a CMIV w/ LAA clip placement. TEE shows a small pouch with clot in the appendage. What happened?

A

The clip was not placed at the base of the appendage.
It should be placed under direct visualization, otherwise a small pouch can be a source of thrombus.

260
Q

In CMIV, what is an alternative to LAA clip?
What are some risks with this?

A

LAA amputation.
This will not remove macro-reentry circuits at the base. If this is done, you must anchor the base to the PV isolation.
Can cause bleeding through the suture line or from tributaries at the AV groove near the coronary sinus.

261
Q

During CMIV procedure, after securing Atriclip or amputation of the LAA, the patient is noted to have EKG changes with T wave inversions in the lateral leads. What are you concerned about?

A

Injury to L circumflex coronary artery causing MI at the time of LAA isolation.
This occurs if the LAA orifice is unusually low (normally at level of frenulum b/w L PV several cm away from MV annulus), and the L Cx is unusually high (normally in the AV groove slightly below the coronary sinus).

262
Q

During CMIV, after completion of the RA appendage line, you come off CP bypass and notice the pt in complete heart block. What happened?

A

Damage to the AV node - cryoprobe too posterior.
Damage to the SA node - prevent by placing the final RA lesion from the distal end of the vertical atriotomy to the tip of the RA appendage. Go as far anteriorly as possible to avoid the pacemaker complex.

263
Q

What patients may be candidates for a stand alone CMIV?

A

In patients who meet all 3:
- failed or are intolerant to antiarrhythmic drug therapy
- failed catheter ablation in the EP laboratory
- feasible minimally invasive approach

It is reasonable to attempt this procedure.
Principle: failed other options and can get min invasive.

264
Q

A patient w/ MR requiring open surgery (TV is nl) has AF as well. Discuss the evidence for PVI (L side only) vs full CMIV.

A

In a series of 305 patients, there was an equal efficacy of a left-only approach, but a higher pacemaker implantation in those who underwent a biatrial lesion set. Both of these findings were confirmed on a meta-analysis of 2225 patients from 10 studies. The equivalent sinus restoration is supported by a randomized subgroup analysis of mitral valve patients that compared patients undergoing biatrial Mazes with pulmonary vein isolation alone.

There is an increased risk of recurrent AF with PVI only.

265
Q

A patient w/ MR requiring open surgery (TV is nl) has AF as well. Discuss the evidence for PVI (L side only) vs full CMIV.

A

In a series of 305 patients, there was an equal efficacy of a left-only approach, but a higher pacemaker implantation in those who underwent a biatrial lesion set. Both of these findings were confirmed on a meta-analysis of 2225 patients from 10 studies. The equivalent sinus restoration is supported by a randomized subgroup analysis of mitral valve patients that compared patients undergoing biatrial Mazes with pulmonary vein isolation alone.

However, in a 2008 analysis of 1723 patients, absence of a biatrial lesion set was a predictor of failure at 48 months. This finding was also supported by a 2006 meta-analysis of 5885 patients in 69 studies. It is currently unknown which patients do better with a biatrial set.

Overall, many surgeons suggest PVI alone for pts w/ comorbidities who cannot tolerate a lengthier operation (especially if it is an isolated mitral) and full CMIV for those with persistent or recurrent AF.

266
Q

A patient is undergoing minimally invasive PVI, but thrombus is discovered on TEE. What should you do?

A

Convert to open.

267
Q

A patient s/p Cox Maze decades ago cannot develop sinus tachycardia during exercise. What happened?

A

One should avoid placing an ablation line through the “sinus tachycardia line” at the right atrium. This can lead to an inability to generate an appropriate response to normal exercise. This was a common complication with CMI (30%).

268
Q

Where is Bachmann’s bundle (CMIV surgery)?

A

Medial and superior to the roof lesion - transecting this will delay the arrival of a sinus impulse from the RA to the LA.

269
Q

What is the “flutter line” and “isthmus lesion”?
Why should it be avoided during CMIV surgery?

A

It is across the cavo-tricuspid isthmus (CTI) - damage results in the inability to develop appropriate bradycardia when asleep.

That said, in patients with cavotricuspid isthmus (CTI)-dependent atrial flutter, ablation along the CTI may be necessary (pic).

270
Q

Define severe TR?
What will symptoms be?

A

Vena contracta width ≥0.7 cm.
Regurgitant volume ≥45 mL.
Central jet ≥50% RA.
ERO ≥0.40 cm2.
Dense continuous wave signal with triangular shape.
Hepatic vein systolic flow reversal.

These pts develop Dilated RV and RA w/ elevated RA with “c-V” wave. Symptoms include dyspnea on exertion, fatigue, ascites, edema.

271
Q

Which patients with progressive TR (stage B) can get surgical intervention?

A

Stage B: Central jet <50% RA, Vena contracta width <0.7 cm, ERO <0.40 cm2, Regurgitant volume <45 mL.
MUST be during L side valve procedure.
AND have annular dilation >4.0 cm OR prior RHF.

TV surgery for this pt has a 2a indication.

272
Q

What are indications for surgery for severe TR?
Differences in recommendations b/w primary and secondary?

A

Only level 1 indication is during L side valve surgery.

If RHF (symptomatic; stage D):
- primary TR has 2a indication.
- secondary TR surgery if: poor GDMT response AND annular dilation WITHOUT inc PAP (2a).
- prior L side valve surgery WITHOUT PH or severe RV dysfx has 2b indication.

If asymptomatic (stage C): primary TR w/ progressive RV dilation or sys dysfx has 2b indication.

Severe TR: Central jet ≥50% RA, Vena contracta width ≥0.7 cm, ERO ≥0.40 cm2, Regurgitant volume ≥45 mL, Dense continuous wave signal with triangular shape, Hepatic vein systolic flow reversal.

273
Q

During TV surgery, where will the AV node be?

A

Triangle of Koch

274
Q

In which direction does the tricuspid annulus dilate?

A

Septal is fixed. Will dilate posteriorly more than anteriorly.

275
Q

How should the tricuspid valve be addressed if taking to the OR for a L valve?
IE what is the order of the surgery?

A

Do L side lesions first.
Tricuspid can be addressed with the heart beating (PFO must be closed before unclamping).

276
Q

Describe a tricuspid ring annuloplasty.

A

Interrupted mattress sutures with a gap a Koch’s triangle (1cm from anterospetal commissure to midpoint on septal leaflet).

277
Q

During tricuspid annuloplasty, what can be injured when placing sutures near the anteropostero commisure?

A

RCA.

278
Q

How do you select ring size for tricuspid annuloplasty?

A

Measure the septal leaflet and surface area of leaflet tissue arising from the anterior papillary muscle.
Usually 30-32 mm for female, or 32-34 mm for male.

279
Q

What are the major principles for tricuspid replacement?

A

Preserve native leaflets, similar to mitral (unless infected).
When placing sutures near the AV node, place them through the septal leaflet instead.

280
Q

What are the principles of the surgery for tricuspid endocarditis? What are the decision making points?

A

If not extensive damage, can excise vegetations to healthy tissue. Close gaps primarily or with a patch.
Consider annuloplasty if valve incompetence is in question.

Replace the valve if extensive destruction.

2-stage replacement can be considered in IVDU with normal RV fct and PAP (fallen out of favor).

281
Q

Benefits of tricuspid bioprosthesis over mechanical?
How does it compare to L side bio valves?
All other things being equal, which would you choose (bio vs mech tricuspid) if you expect heart block?

A

No anticoagulation.
Less structural valve deterioration than those in mitral position.
Does not limit transvalvular PM leads in the future.

Be careful placing Swan postop.

282
Q

When coming off pump following TV replacement, the RV dilates, CVP is high, and PAP decreases. How do you manage?

A

Rule out metabolic issues and air emboli.
TEE for assessment.
Eval RVOT for obstruction: eg, anterior leaflet tissue billowing -> central portion can be excised while maintaining chordal attachments.
Eval for ST changes from RCA occlusion (inferior leads; caused w/ deep stitch at ant/post commissure) -> bypass RCA w/ vein.
Make sure there’s a Swan-Ganz that’s surgeon directed after this procedure (cannot do if mechanical).

If above ruled out, and pt has RHF, may not be able to come off. Transition to ECMO. You would expect CVP and PA pressures to go up some because the “popoff” that TR creates is now gone, and the RV is having to push all blood through the pulmonary system.

283
Q

Manage RV dysfunction postop TV repair?

A

Similar principles to other RV failure:
Improve hypoxia and decrease hypercarbia. Decrease pressure in lungs.
Support contractility/inotropy w/ Epi, increase lusitropy (and inotropy) w/ Milrinone.
Restrict fluids. Diurese.
Can add nitric to vent or HFNC to unload PA more w/ less systemic effects than milrinone.
Can use IABP to unload even more and improve RCA perfusion.

284
Q

When should a tricuspid valve be done in a reoperative patient?

A

Principle: it has to be bad, and it can’t be complicated:
- Stage D (Central jet ≥50% RA, Vena contracta width ≥0.7 cm, ERO ≥0.40 cm2, Regurgitant volume ≥45 mL, dense continuous wave signal with triangular shape, Hepatic vein systolic flow reversal; AND RHF symptoms).
- No PH or RV systolic dysfunction.

285
Q

Coming off CPB from TV surgery, the patient has complete heart block.
What happened?
What do you do?

A

Sutures near the anteroseptal-septal commissure can easily damage the AV node.
Radial force from a prosthetic valve (or dual prosthetics in the MV and TV position).
2 sets of V pacing wires would not be overkill; in addition to external pacing pads before eventual PPM placement.

286
Q

What are options for pacemaker placement after tricuspid surgery?

A

Endocardial - percutaneous into atrium, then coronary sinus.
- if unable to get into coronary sinus, can traverse repaired valve.
Epicardial - avoids foreign object through repair/replacement. Only option for mechanical valve.

287
Q

Pt w/ TV endocarditis (+ cultures, and vegetations) has previous PPM, and has an indication for mitral surgery. How do you handle the pacer leads?

A

Remove the initial wires in setting of bacteremia and endocarditis. New epicardial leads on the RV can be placed (tunneled through the L intercostal space away from the mammary and out to a subcutaneous pocket). Transvenous atrial wires can be placed later if needed or if bacterial burden is too high on initial surgery.

288
Q

In carcinoid valvular disease, what is the gross appearance and management of an affected TV?

A

Grossly, you will see white fibrous tissue deposits/plaques on the endocardium of valve cusps on the ventricular side, causing adherence to the RV wall.
TV replacement required. Check pulmonic valve - may need to be replaced as well.
L heart valves are not affected d/t inactivation by MAO as it passes through the lungs.

289
Q

What’s the best study for tricuspid valve?

A

Preoperative TTE. This is where you will make your decisions.

290
Q

Before performing TV repair with a beating heart, what anatomic pathology must be ruled out?

A

PFO.
Otherwise air gets into the L side.

291
Q

A patient who is undergoing left side valve surgery gets a LHC preop. What is your cut off to bypass a lesion? What about LM lesions?

A

70% any lesion.
50% LM.

292
Q

You are coming off bypass from an AVR CABG w/ a 2 hr cross-clamp time. CI is 1.7. What do you do to assess? What do you do if no technical problems?

A

Look at the heart and the EKG.
TEE to eval for MWAs. Assess grafts with flow probe.

Determine if there were issues with cardioplegia - if all above ruled out, it could be cardiogenic shock from poor protection.

Rest the heart on full flow with the heart completely decompressed for 10-15 min.

May require IABP. If the pt worsens over the next 12 hrs, consider catheterization to assess grafts.

293
Q

Sequence of surgery for CABG/MVR/AVR in pt w/ AF?

A

Distals, aortotomy w/ AV resection, CMIV, MVR, AVR, proximals.

294
Q

You are finishing an AVR/MVR when TEE shows lateral WMA.
What happened?
What do you do?

A

You’ve injured the circumflex likely during the placement of posterior leaflet mitral valve sutures.
Re-heparinize and go back on bypass. Take vein, and bypass the circumflex.
Keep in mind that you have a new mitral that could be easily dislodged, or you could cause an AV groove disruption when exposing the lateral wall.

295
Q

What complications could oversizing the mitral valve cause?

A

LV rupture when the heart is contracting after coming off CPB.
Higher risk of block requiring PPM.
If concurrent with AVR, then oversizing MV can cause downsizing of the aortic valve -> pt prosthesis mismatch.

296
Q

A patient has severe AS and severe MR. He is induced and subsequently arrests.
What happened?
What do you do?

A

SVR drops, venous return decreasing, coronary flow decreases, ventricles fail.
The incidence of arrest on induction is higher for combined pathology than for isolated severe AS.

Secure the airway. Ventilate. Begin CPR/ACLS. Heparinize. Prep/drape/scrub. Emergency sternotomy and quick cannulation. Initiate CPB. Proceed with AVR/MVR.

If truly worrisome preop, consider awake line placement, and prep and drape before induction w/ sheaths in the groin and a sternal saw working.

297
Q

What important structure is behind p2-p3 of the mitral valve?

A

Circumflex artery

298
Q

After an AVR/MVR, you are coming off bypass and see bright red blood coming from behind the heart. What do you do? (What do you NOT do?)
What increases risk for this complication?

A

Do NOT lift the heart. This is likely AVG disruption. Go back on bypass first, then repair - depending on location, can do an internal patch repair, external primary repair with pledgets or strips.

Risks: extensive annular decalcification near the posterior leaflet, use of an oversized prosthesis, or excessive traction on the left ventricle

299
Q

What is the conduct of an AVR/MVR?

A

Open aorta, excise leaflets and debride annulus, explore mitral and *do NOT oversize if requiring replacement (size aortic valve at the same time to determine if adequate AV prosthetic will fit), replace mitral, re-size aortic valve, replace aortic valve.

*If mitral is oversized before you fit AV, you may end up with prosthesis mismatch.

300
Q

During AVR/MVR, what structure is near mitral lateral A3 and the membranous septum of the aortic valve near the R and non-coronary cusps?

A

Conduction system.
Don’t oversize valves - can compress this system.
Place A and V wires on all double L valve cases.
PPM risk is 5% for single valve. It’s 15% for >1 valve.

301
Q

A pt undergoes AVR/MVR. You come off bypass, and TEE shows the following, with increased gradient across the LVOT. What happened?

A

MV strut is directed into the LVOT. Re-replacement of the valve should be done. This is more likely with bioprosthetic valves. These valves usually have markings for their struts that allow the surgeon to orient the valve in such a way that the space between the struts is over the LVOT.

302
Q

In pt with significant multivessel CAD and significant carotid disease, how do you decide which to approach first?

A

Most symptomatic should go first with a 4-6 waiting period before second surgery. If both are symptomatic, can do synchronous procedure.

303
Q

After a combined CEA and CABG, the patient has neuro deficits associated with operative CEA. What do you do?

A

If immediately postop, would get bedside ultrasound evaluation of flow and return to the OR if flap or technical problem is found. If out of PACU, would get head CT and carotid duplex.

304
Q

After a combined CEA and CABG, the pt develops a large neck hematoma with tracheal deviation. What do you do?

A

If time permits, the patient should go to the OR, prep and drape, intubate, then reopen.
If not, open at bedside and be ready to hold pressure while you get to OR and intubate.

305
Q

What are the major Duke criteria for endocarditis?
Positive is 2 major, 1 major and 3 minor, or 5 minor.

A

Major: microbio (atypical requires time b/w 2+ cxs or 3/4+ cxs); endocardial involvement (new regurg, abscess, vegetation, dehiscence of prosthetic).

306
Q

What are the MINOR Duke Criteria?
Are there minor echocardiographic criteria?

A

Predisposition – Intravenous drug use or presence of a predisposing heart condition (prosthetic heart valve or a valve lesion associated with significant regurgitation or turbulence of blood flow).
Fever – Temperature ≥38.0°C (100.4°F).
Vascular phenomena – Major arterial emboli, septic pulmonary infarcts, mycotic aneurysm, intracranial hemorrhage, conjunctival hemorrhages, or Janeway lesions.
Immunologic phenomena – Glomerulonephritis, Osler nodes, Roth spots, or rheumatoid factor.
Microbiologic evidence – Positive blood cultures that do not meet major criteria, OR serologic evidence of active infection with organism consistent with IE.

*There are no echocardiographic minor criteria.

307
Q

How can the diagnosis of infective endocarditis be rejected (4 scenarios)?

A

A firm alternate diagnosis is made.
Resolution of clinical manifestations occurs after ≤4 days of antibiotic therapy.
No pathological evidence of infective endocarditis is found at surgery or autopsy after antibiotic therapy for four days or less.
Clinical criteria for possible or definite infective endocarditis is not met.

308
Q

What are the positive echocardiographic findings for infective endocarditis?
(and thus fulfill a major Duke criteria)

A

-Vegetation (oscillating intracardiac mass on a valve or on supporting structures, in the path of regurgitant jets, or on implanted material, in the absence of an alternative anatomic explanation).
-Abscess.
-New partial dehiscence of prosthetic valve.
-New valvular regurgitation.

309
Q

What are the principles of empiric antibiotic therapy in suspected infective endocarditis?

A

For patients with suspected IE who present WITHOUT acute symptoms, empiric therapy is NOT always necessary, and can be deferred until blood culture results are available, particularly since accurate microbiologic diagnosis is a critical first step in planning the treatment strategy.

For acutely ill patients with signs and symptoms strongly suggestive of IE, empiric therapy may be necessary.
Such empiric therapy should be administered only after at least two (preferably three) sets of blood cultures have been obtained from separate venipunctures and ideally spaced over 30 to 60 minutes.

310
Q

What should empiric antibiotic therapy cover in IE?

A

In general, empiric therapy should cover staphylococci (methicillin-susceptible and methicillin-resistant), streptococci, and enterococci.

311
Q

When should a TTE or TEE be repeated in treatment of infective endocarditis?

A

Clinical indications for repeat TTE and/or TEE include new murmur, embolic complications, new or progressive heart failure, and development of atrioventricular block. In addition, patients with persistent fever or persistent bacteremia (eg, longer than five to seven days) warrant repeat echocardiography to evaluate for abscess.

312
Q

What is the role for ASA or antithrombotic therapy for infective endocarditis?

A

neither anticoagulant therapy nor aspirin reduces the risk of embolism in patients with IE; remember that pts w/ IE are also at increased risk for bleeding complications (particularly intracerebral)

313
Q

In pts w/ IE, what atrial fibrillation pts require anticoagulation no matter the CHA2DS2-VASc score?

A

mitral stenosis

314
Q

What IE pts need to be screened with dental evaluation?

A

IE due to common oral organisms. All active sources of oral infection should be eradicated.

315
Q

What further screening is needed if IE is due to group D streptococci?

A

colonoscopy is warranted - association w/ cancer

316
Q

When should IE on R side valves receive surgery?

A

Indications for surgery in R NVE include:
- very large vegx (≥20 mm)
- recurrent septic pulmonary emboli
- highly resistant organisms
- persistent bacteremia

317
Q

When should early valve surgery be performed for L side IE?

A

*Heart failure (HF) symptoms - benefits of surgery appear to be greatest among patients with severe valve dysfunction causing HF, surgery should be undertaken as soon as signs and symptoms of HF appear and before hemodynamic instability occurs.
*Complicated infection - paravalvular extension of infection with development of annular or aortic abscess, destructive penetrating lesion (eg, fistula), and/or heart block
*Difficult-to-treat pathogens - include fungi and multidrug-resistant organisms (eg, Pseudomonas aeruginosa and vancomycin-resistant Enterococcus). NOT S. aureus.
*Persistent infection - bacteremia or fever lasting >7 days after initiation of appropriate antibiotic therapy.

318
Q

For neuro symptoms associated with IE (in pt w/ indication for surgery), should surgery be delayed?

A

Depends on how severe and what kind of stroke.

No delay: silent microembolism, transient ischemic attack, cerebral abscess, or ischemic stroke with no hemorrhagic conversion and without severe neurological impairment or decreased level of consciousness.

For patients with major ischemic stroke (ie, with severe neurologic deficit or altered consciousness), or intracranial hemorrhage, many would delay surgery for ≥4 weeks.

319
Q

In the setting of active infection in a pt w/ IE and a surgical indication, is repair or replacement preferred?

A

Valve REPAIR is preferable to valve replacement when feasible; valve replacement is associated with a small risk of infection of prosthetic materials. Valve repair may be possible when a leaflet perforation occurs in the absence of extensive leaflet destruction or annular involvement.
Also, there is no increased reinfection risk b/w mechanical and bioprosthetic valve installed after IE.

320
Q

How does a prosthetic valve affect the decision for surgery for IE?

A

Early surgery for prosthetic valve endocarditis is indicated for IE complications SIMILAR to native valve endocarditis (eg, HF due to prosthetic valve dysfunction, paravalvular regurgitation, or intracardiac fistula; annular abscess, difficult-to-treat pathogen, persistent infection).

321
Q

You are doing preop work for AV IE and find an annular abscess at the R/non-con commissure. What do you do in the OR?
What complications should you watch for?

A

Completely *unroof the abscess and debride grossly infected tissue.
*Patch the defect w/ autologous or bovine pericardium.
Involvement of a large part of the annulus may require replacement w/ *homograft/xenograft (muscular portion may fill in tissue defect after extensive debridement).

The R/non-con is the site of the *membranous septum, so VSD can occur w/ aggressive debridement.
R/non-con is also near the *conduction system, so inc risk for conduction disturbance and PPM need.

322
Q

You are doing surgery for IE on mitral valve. Only a part of P2 is involved. What should be done?

A

Excise and repair. Avoiding placement of prosthesis i preferred in an infected setting.

323
Q

You are doing surgery for IE on mitral valve. The entire posterior annulus is involved with abscess.

What should you do?

A

Careful debridement of all tissue. If enough tissue remaining, reinforce the posterior annulus w/ interrupted figure of 8 sutures from LA to LV, creating a fold of tissue which can be used to secure the prosthetic valve.

Be careful of the coronary sinus, circumflex, and bundle of His.

324
Q

Common causes of culture negative endocarditis?

A

HACEK: Hameophilus, Actinobacillus, Cardiobacterium, Eikenella, Kingella.
Also fungal organisms.
Consult ID.

325
Q

Discuss the timing for intervention in L side endocarditis in patient with stroke.

A

Depends on patient risk factors and whether it is embolic or hemorrhagic. Controversial timing.
For hemorrhagic stroke, most agree that operation should be delayed. The duration requires judgment and balancing risks of cardiac vs neurologic complications. Reasonable to delay 3 to 4 weeks after the stroke.
If there is a significant decline in cardiac function, recurrent stroke/systemic embolism, or persistent sepsis, a delay of less than 4 weeks may be necessary.
With embolic CVA in the absence of bleeding, an operation may be safe in 3 to 5 days.

326
Q

Describe management for R side valve endocarditis.
First line?
Most common pathogens?
Indications for surgery?
Other workup?

A

Initial tx is abx.
S aureus 70%. Strep and enterococci are next most common.

Surgery indications: (get TEE in all, get cardiac CT if possible abscess)
- 2cm vegetation
- recurrent PE
- highly resistant organism (MDR enterococci, P aeruginosa, fungi, other MDR GNB, MRSA [not MSSA], Brucella)
- RHF 2/2 severe TR
- heart block
- paravalvular abscess

327
Q

What is the INR goal for tricuspid mechanical valve?
(no additional risk factors, not high-risk valve)

A

3.0. Same as mitral.
Ball-in-cage and tilting disc valves (other than Medtronic) require 3.5 in the mitral or tricuspid position.

328
Q

A non-adherent IVDU patient has second recurrence of tricuspid endocarditis with a surgical indication. What other options are there?

A

valve excision without replacement - if pulmonary pressures and pulmonary vascular resistance are not elevated and good RV and LV fct

329
Q

Describe considerations in management for prosthetic valve endocarditis.

A

Need to know timing and if complicated…
Is it early or late? <2 mo after surgery is often aggressive (staph) and requires intervention within 1-2 weeks. If late PVE (>2 mo; strep), treat like native valve endocarditis.

Is there valve dehiscence or paravalvular leak? Requires intervention within 24-48 hrs.

330
Q

A pt w/ AV endocarditis and paravalvular abscess requires a homograft. The pt is in complete heart block after weaning from CPB. What is the ideal management?

A

permanent epicardial atrial and ventricular epicardial pacing leads are less likely to become infected than transvalvular leads

331
Q

You do an emergency ED thoracotomy for a pt who was stabbed. You place pledgeted sutures. Coming off CPB, there is localized wall motion abnormality and corresponding ST elevation. What do you do?

A

Probably coronary artery injury. Use vein and bypass the injured coronary.

332
Q

A patient is stabbed in the chest and taken to the OR. After sternotomy, you find a RA injury. How do you manage?

A

Clamp the RA and sew. No need for CPB.
Eval the heart with TEE and ensure no ASD/VSD/valvular damage.

333
Q

You discover a large ventricular injury in penetrating trauma. How do you manage?

A

Larger defects may require a *patch. Otherwise pledgeted *horizontal mattress sutures often work.

Make sure *septal injury and *valve injury are ruled out with echo if dealing with large defects or multiple chamber defects.

334
Q

There is penetrating chest injury to the coronary sinus in a sick trauma patient. How do you manage?

A

Ligate it.
Patch can be considered if the patient and heart are not friable.

335
Q

When can you ligate a traumatic coronary injury?

A

If it is distal, and there is not reversible ventricular dysfunction.

336
Q

What is the major mechanical force (pathology) that leads to aortic dissection?
Other factors?

A

Hypertension.
Other risk factors: connective tissue disease, fam hx of aneurysm or dissection, aortic valve dz, pregnancy, recent open or catheter aortic manipulation, stimulants.

337
Q

Where are you likely to find the dissection tear in type A?

A

right anterior aspect of ascending aorta, spiraling around the arch into the descending

338
Q

What are the causes of mortality in typa A aortic dissection?

A

aortic rupture, pericardial tamponade, acute AI and CHF, acute MI, stroke

339
Q

What is the ideal study for a type A dissection?

A

CT angio from neck through femoral vessels, EKG-gated

340
Q

What study can be done to diagnose type A aortic dissection in an unstable patient?

A

If suspicion is high, get TEE on-table in OR.

341
Q

What is your exam and workup for possible type A dissection?

A

Vascular history.
Palpate pulses x4 extremities and carotids. BP in BUE to eval diff.
Neuro exam.
EKG, D-dimer, CBC, lytes, LDH, cardiac markers, coags, type/cross.
CXR - widened mediastinum, single side effusion
CTA EKG gated vs MRA. TEE if cross sectional imaging contraindicated or not available.

342
Q

What d-dimer level makes type A dissection less likely?

A

<500 ng/dL

343
Q

What are the initial goals of management for type A dissection?

A

Access and monitoring.
HR <60 and BP 100-120 systolic.
Esmolol w/ loading dose 250-500 mcg/kg, then 25-50 mcg/kg/min.
Can add nitroprusside at 0.25-0.50 mcg/kg/min if esmolol controls HR, but BP needs control.
Pain control.
Foley.

344
Q

What are the cannulation strategies for a type A dissection undergoing surgery?

A

Ensure good CT review to know where dissection is and isn’t. Know what extent of dissection is to know how much of arch needs to be done.

R axillary artery w/ graft.
Direct cannulation of ascending aorta: Seldinger w/ TEE guidance vs samurai.
Femoral artery.
Venous cannulation: usually dual stage, but bicaval can be done if retrograde cerebral perfusion needed.

345
Q

CTA shows type A aortic dissection with arch vessel involvement. What surgery should be done?

A

Hemiarch is often sufficient to completely resect the tear and exclude the false lumen to re-establish arch vessel perfusion.

If the actual TEAR involves the arch vessels (not just the dissection), the arch may need to be replaced.

346
Q

In type A dissection what would necessitate more extensive ARCH surgery?

A

Arch rupture.
>5.5 cm arch aneurysm.
Tear involves arch.

347
Q

In type A dissection, what are the options if the aortic root is involved?

A

Bentall, David.

348
Q

Type A dissection found in pt under 40 yrs.
What should you evaluate for?
Why?

A

CTD - Marfan, Loeys-Dietz, Turner (female).
If CTD is known or suspected, the ENTIRE ROOT should be replaced.

349
Q

In type A dissection w/ CTD, future reinterventions are most frequently due to failure to do what?

A

failure to replace the whole root, failure to replace the entire ascending aorta, or aneurysm of the aortic wall at the coronary artery button

350
Q

How do you manage intramural hematoma in the ascending aorta?
How do you diagnose an IMH?

A

Treat like ascending dissection.
CT IMH hallmarks: absence of intimal tear, absence of false lumen flow (differentiates from penetrating aortic ulcer).

351
Q

In type A dissection, how do you manage dissection of coronary ostia?
Which his more commonly involved L or R ostia?

What do you do if only the ostia are involved?

What if the dissection tears into the coronary vessel itself?

A

RCA is more frequent than left, but both need to be evaluated.
May need retrograde cardioplegia.

If only ostia - mobilize the buttons and reapproximate the dissected layers.

If the coronary vessel is dissected - bypass or interposition with GSV.

352
Q

A type A dissection patient is going on bypass via central cannulation. The femoral arterial line monitor shows a decrease in pressure. What happened? What do you do?

A

False lumen compression.
It’s possible that either the femoral art line OR the arterial bypass cannula is in the false lumen.
Check a radial pressure. If it’s normal, then the femoral is either malperfused (possibly re-entry tear) or the femoral line is in the false lumen. Either way, it’s unreliable.
If radial pressure is dampened too, then check the TEE, but plan to recannulate.

Due to re-entry tears, distal flow dynamics can change with changes in flow. Look for this to happen on initiation of CPB, cross clamping, or after circ arrest.

353
Q

In a type A dissection, SMA occlusion is found. How do you manage?

A

Often, re-establishing true lumen flow at the type A will be enough.
If there is a question about bowel integrity, then do a laparotomy and assess.
Stenting, fenestration, or bypass may be needed.

354
Q

A patient with type A becomes comatose after decerebrate posturing (preop). What do you do?

A

Patients with dense neurological deficits, have an extremely poor prognosis. It may be best not to offer surgery.

In those with less severe neuro deficits, even paraplegia, surgical repair is warranted, all else being equal.

355
Q

If there is ileofemoral malperfusion during a type A dissection, what should be considered for intervention?
What if the pulse has been lost for a few hours?

A

Ax-fem or fem-fem bypass.
If no flow for 6 hrs, may require ppx fasciotomies (including operative time), especially if there is outflow/venous compromise, unless you can perform careful and frequent serial exams, including compartment pressure evaluation with a trigger for fasciotomy at delta pressure <30 (ie compartment less than 30 mmHg below diastolic blood pressure)

356
Q

How do you manage a chronic type A aortic dissection?

A

Defined as >90 days.
Scenario is a CT done for another reason with the patient remembering an episode of severe tearing back pain a few months ago.

Surgery is indicated for complications or symptoms, eccentric expansion, rapid expansion, or aortic insufficiency.
Get an echo, a good physical exam, including BUE and lower extremity BP, and a good review of symptoms, including myocardial ischemia symptoms.

357
Q

A patient s/p type A dissection repair shows distal false lumen perfusion. What are they at risk for? How do you manage?

A

Risk: Aneurysmal dilation and rupture.
Mgmt: strict BP control to <120 systolic.
CT surveillance at 1, 3, 6, and 12 months is recommended. Yearly after this.
70% will be free from reoperation at 10 years. Echo to eval the aortic valve at some point is warranted.

358
Q

How do you manage a type B dissection?

A

1) Medical. BP and HR control (impulse control). Usually esmolol drip w/ nitroprusside if needed.
Malperfusion treated as needed.

2) Look for complicated disease or high-risk anatomical features that would push for intervention.

359
Q

You are starting a CABG, and the arterial cannula is placed. There is high line pressure, and the aorta develops an enlarged purple hematoma. What happened? How do you confirm?

A

This is an aortic dissection.
Adventitial hematoma could be the cause, but the high line pressure makes this dx less likely.
Confirm w/ TEE.

360
Q

How will an iatrogenic (intraop) ascending aortic dissection manifest? What should you assess?

A

Increase in line pressure after arterial cannula placement. Purple hematoma involving the ascending aorta. MAPs drop.
TEE to confirm via a flap in the ascending aorta.
Eval EKG for ST changes. Eval pericardial well for blood.

361
Q

Manage an iatrogenic ascending aortic dissection.

A

Stop perfusing through the cannula.
Treat hypotension w/ volume and pressors.
Hypotension, if it occurs, in this setting is likely 2/2 myocardial ischemia or acute AI.
There is no tamponade w/ the pericardium open, and rupture should be visualized.

Cannulate the axillary or femoral artery.

Clamp higher, evaluate coronaries, aortic valve, previous clamp site. Find tear. Hopefully it is contained within ascending aorta. Replace if so. If not, may need arch surgery.

362
Q

During surgery, you cause an iatrogenic ascending aortic dissection. The LV begins to distend. What does this mean?

A

Aortic valve incompetence.
Place an LV vent or open the aorta.

363
Q

You are replacing the descending thoracic aorta for a type B. During systemic cooling, the LV distends, and VF occurs. What do you do to decompress?

A

Decompress via the L SPV or LV apex.

364
Q

How can aortic dissection occur at the end of the case?
What do you do?

A

Removal of the cross clamp under pressure. Make sure “flows way down” before removing clamp.
If this occurs, re-heparinize, and obtain axillary or femoral cannulation.

365
Q

How do you manage acute AI in the setting of iatrogenic ascending aortic dissection?

A

Evaluate the leaflets. If they are intact, repair the aortic dissection and resuspend the aortic valves as long as anatomy allows you to realign the center of coaptation.
Reevaluate the valve w/ TEE before leaving the OR.

366
Q

S/p CABG, a pt develops signs of MI and hemodynamic instability in the ICU. What is your workup?

A

EKG to eval for ST changes in specific anatomy.
CXR to eval for bleeding.
TTE to eval for WMA and possibly dissection.

If likely coronary problem, proceed to angio.
If likely aortic dissection, get gated CTA if stable or TEE in OR.

367
Q

You are attempting a mini-thoracotomy cardiac surgery requiring bypass. You cannulate the femoral, but resistance is noted. What is the next step, and what are you worried about? What if the dx is confirmed?

A

If concerned, can do an angio to eval for dissection.
If type B dissection is confirmed, check TEE to eval for ascending. Abort the procedure, do CTA to confirm extent. Medically manage for 4-6 wks and return to OR later. Cannulate a different site.

368
Q

What are the indications for repairing a TAA?

A

Symptomatic (aneurysm defined at 4 cm).
Ruptured.
End-diastolic diameter 5.5 cm.
Aortic size index 2.75 cm/m2.
Aortic area over height ratio 10 cm2/m.
If undergoing AV surgery or CABG, then EDD 4.5 cm.
If getting surveillance, expansion ≥5 mm per year for aneurysms <5 cm in diameter.
If bicuspid aortic valve, 5 cm.
Genetic factors also affect size cutoffs (image).

369
Q

How does bicuspid aortic valve affect indication for surgical repair in aortic root aneurysm?

What about AS or AR? Any changes in indication for intervention?

A

No changes for ascending aortic aneurysm (5.5 w/o additional risk factors, 5 for increased risk or high-volume centers, 4.5 if other cardiac surgery indication).

No changes for AS/AR. Standard recommendations for aortic valve intervention apply to patients with bicuspid aortic valve.

370
Q

In pt w/ aortic root aneurysm requiring surgery, how do you manage the valve operatively?

A

If valve is normal, a valve sparing procedure can be performed - David.
If <40 yrs old, consider Ross.

371
Q

You finish replacing the aortic root and are deairing. There is 11mm ST elevation in the inferior leads (II, III, AVF). How do you manage? How do you prevent this?

A

Consider technical problem with the RCA button. The change in orientation can cause kinking, stretching, or compression.
Bypass the RCA.
Prevent this by sizing and placing the button with the heart engorged.
Can also manifest as difficulty coming off bypass, new WMA, arrhythmias, new EKG changes, or unexplained RV failure.
DDx is poor myocardial protection, air embolism, protamine reaction.

372
Q

You do a valve sparing root replacement and are coming off bypass. There is 3+ AI. What do you do?

A

Go back on, resect the valve, and replace it.

373
Q

When coming off CPB after root surgery, there is persistent blood coming from behind the proximal anastomosis. What happened?

A

Probably bleeding from the root vs LCA suture line.
Pack it and use topical agents. Stop the rest of the bleeding in the surgical field. If the bleeding persists, you may need to go back on CPB and place the stitch after the heart is arrested and emptied.
If the pt cannot tolerate another pump run, pack the chest open, resuscitate, and go back.

This is a difficult suture to place once the patient is off CPB. You can injure the LA dome. Prevent it by taking your time on the posterior portion of the aortic anastomosis.

374
Q

How do you follow aortic root patients after replacement?

A

BP control and anticoag mgmt (if valve surgery done). Postop CT is needed to assess growth of non-replaced aorta and to eval for aneurysm formation.

375
Q

How are pts w/ aortic root dilation that don’t meet surgical criteria treated?

A

Control HTN. Optimize lipid profile. Smoking cessation. Reduce risk factors for atherosclerosis.

376
Q

You are doing a redo aortic root replacement, but cannot safely dissect the tissue around the aortic root posteriorly. What are your options?

A

The implication is the coronary arteries will be difficult/dangerous to mobilize.
Cabrol technique - coronary reimplantation by placing an interposition tube graft to the open coronary ostium then doing a side-to-side anastomosis to the main aortic graft.
Hemi-Cabrol - direct implantation of the RC button (usually easer to mobilize) and reimplantion of LC artery w/ interposition graft b/w the LC ostium and aortic graft.

377
Q

Diameter indication for surgical repair of sporadic ascending/root aortic aneurysm?

A

5.5 cm.
5 for Marfan, Ehler-Danlos, or with experienced surgeons.

378
Q

Diameter indication for surgical repair of sporadic aortic arch aneurysm?

A

5.5 cm.
There isn’t a 5.0 or 4.5 caveat here, unlike ascending aneurysms.

379
Q

Diameter indication for surgical repair of Marfan associated ascending aortic aneurysm? Anything else to consider?

A

Need to know if risk factors for aortic complications - FHx of dissection, progressive aortic regurgitation, rapid expansion (increase by 0.5 cm or more per year).
- no risk factors: 5 cm
- risk factors: 4.5 cm

*these cutoffs include TAA assd w/ following: familial thoracic aortic aneurysm/dissection, Turner syndrome, and other non-BAV congenital conditions. It is unclear when to intervene for EDS; some suggest surgery for acute events only.

380
Q

For those with rapid expansion of an ascending TAA, what is considered “rapid,” and when should intervention be done?

A

Rapid: increase by 0.5 cm or more per year. 0.3 cm in 2 years.
A lower diameter may be warranted (eg, 5.0 cm for sporadic TAA).

381
Q

Diameter indication for surgical repair of Loeys-Dietz ascending aortic aneurysm?

A

Need to know if risk factors for aortic complications - FHx of dissection, progressive aortic regurgitation, rapid expansion (increase by 0.5 cm or more per year). For LDS, additional risk factors include female sex and TGFBR2 mutations.
- none: 4.5 cm
- risk factors present: 4 cm

382
Q

Diameter indication for surgical repair of sporadic ascending aortic aneurysm associated w/ bicuspid aortic valve?

A

Same as for standard ascending.
Need to know if risk factors for aortic complications - FHx of dissection, progressive aortic regurgitation, rapid expansion (increase by 0.5 cm or more per year).
- none: 5.5 cm
- risk factors present: 5 cm
- severe AI or AS: 4.5 cm

383
Q

In a patient undergoing cardiac surgery for another reason, at what diameter should you add intervention for an ascending aortic aneurysm?

A

4.5 cm

384
Q

What is the law of laplace and how does it apply to TAA?

A

tension = pressure x radius
These aneurysms will continue to dilate, though smaller sized aneurysms can stay stable over a long period. >6 cm have significant increase in mortality.
Repair usually indicated at 5.5 cm if asymptomatic. Can be smaller depending on other conditions/risk factors.

385
Q

Characteristics of Loeys-Dietz syndrome?

A

Mutation in TGF-B receptor.
Hypertelorism, bifid uvula, arterial tortuosity. Aneurysms.

386
Q

Characteristics of Ehlers-Danlose syndrome

A

Autosomal disorder of COL3a1 gene for type III collagen.
No formal aneurysm guidelines as surgery is very high risk. Often present w/ arterial rupture.

387
Q

What infection is assd w/ TAA?

A

Syphilis - infection destroys muscular and elastic fibers of the media layer.

388
Q

In a patient with previous ascending aortic dissection, what chronic aortic disease process may present later?

A

Aneurysm. The uncorrected portion of the aorta tends to continue to dilate. It is often at a faster growth rate with a higher risk of spontaneous rupture.

389
Q

What workup do you want in an ascending TAA pt for possible surgery?

A

CTA or MRA - extent of aneurysm, include carotids; look at root and arch; include abd/pelvis if considering femoral cannulation; look at proximity to sternum.
Echo - eval valves, cardiac function.
Cardiac cath - eval coronary vessel disease.
PFTs - if smoker w/ COPD; helps determine risk.
CBC, CMP, Coags, T/C.

390
Q

What are the cannulation considerations in an ascending aortic aneurysm surgery?

A

Cannulate and clamp higher than what needs to be replaced.
Can do central if there’s room for cannula, clamp, and sutures.
If no room, can do axillary.

391
Q

During ascending TAA repair, what can happen if you anastomose an aortic graft that is too long?
How do you perform anastomosis?

A

It can kink when the heart fills.
Distal first. Use felt on aorta if tissues are bad. Move clamp on to graft to eval for hemostasis.
Proximal anastomosis. Place root vent in graft prior to cross clamp removal and turn off vents while suturing to help deair.
Postop BP control. DC w/ beta-block, ARB, and statin.

392
Q

Planning for ascending TAA repair. What if you cannot clamp the ascending aorta with an adequate sewing cuff?

A

May need circ arrest.

393
Q

You are doing an ascending TAA repair. You fill the heart, and the graft kinks. What do you do?

A

Can attempt to plicate the lesser curve, but may need to cross clamp, arrest, excise excess graft, and sew graft to graft.

394
Q

You start an ascending TAA repair, but the LV distends once commencing bypass. What is happening?

A

Significant aortic insufficiency may be present. This should be expected based on preop eval.
Clamp and arrest w/ retrograde.
Decompress with antegrade vent. If one is not in place, transect the aorta, and deliver direct coronary cardioplegia.
Replace the valve.

395
Q

You are starting surgery for ascending TAA, and during the sternotomy, there is a large rush of blood. You think the aorta ruptured. What do you do?

A

Give heparin. CLOSE the sternum w/ towel clamps.
Tell anesthesia. Get blood in the room.
Get perfusion ready for cannulation.
Get femoral access in case you need to cannulate there.
Give heparin.
Open the sternum and place manual pressure on the hole. Get on bypass. Use lots of suctions for bypass.
Cool. Clamp the aorta above the rupture. Open it and arrest the heart.
Place an LV vent and retrograde sinus cannula if needed.

If aneurysm at high risk preop, may be better to axillary/femorally cannulate and be on bypass first. If risk very high, may even cool to 18 degrees for circ arrest before sternotomy.

396
Q

What are the size cutoffs for intervention in patients with an arch aneurysm compared to ascending aneurysm?

A

The same for the cutoff for sporadic disease. Ascending aorta has caveats for centers of excellence for 5.0 cm cutoff.

397
Q

What are the options for aneurysmal arch repair, and what does the choice depend upon?

A

Hemiarch, total arch, elephant trunk. Decision depends on distal extent of aneurysm.

398
Q

For the different types of aneurysmal aortic arch repair, what cannulation strategy will work for almost any situation?

A

R axillary artery; unclutters the field and allows for selective ACP

399
Q

How can you achieve brain perfusion for an uncomplicated hemiarch for arch aneurysm? What if the repair is more complicated or extensive?

A

retrograde via SVC usually works;
antegrade brain protection may be needed for more extensive repair;
these methods extend the safe period (ie lower mortality/stroke) for circulatory arrest, selective ACP is just as safe as bilateral ACP; selective ACP extends the safe time more than retrograde CP.

400
Q

How do you initiate antegrade brain perfusion (arch aneurysm repair)?

A

After clamping and arresting, cool to 18-20 degrees and until EEG is silent.
Give pentobarbital and mannitol.
Turn off CPB.
Transect aorta.
Clamp the R innominate, and initiate antegrade brain perfusion: 500 ml/min (10-15 ml/kg) w/ mean pressure 40-60 mmHg and good backflow through L CCA.
If not good flow/backflow, place a perfusion catheter up L CCA and Y into the antegrade perfusion circuit at similar rate.
Finish the surgery.

401
Q

When do you need a total arch repair for arch aneurysm?

A

When aneurysm extend into the mid/distal arch (past L CCA).

402
Q

When doing a total arch repair for aortic arch aneurysm, how do you decide to manage the head vessels?

A

If they are aneurysmal, they may need reconstruction. If not, can be fashioned to the graft as an island.

403
Q

What is the safe time period for circulatory arrest at 18 degrees?

A

The relatively safe time period for circulatory arrest at 18°C is 25 to 30 minutes.

404
Q

When do you add an elephant trunk to aneurysmal arch repair?

A

When the distal arch or isthmus is involved. Only if a seal can be created. If not, conventional approach should be done.

405
Q

On preop imaging for an aneurysmal arch surgery (total arch), an aberrant LVA is found. How does this change the surgery?

A

Duplex eval of the RVA to ensure patency and antegrade flow. Eval for intact circle of willis.
If RVA is ok, LVA can be temporarily occluded, and reimplanted during patient rewarming (into L CCA, SVG to aorta, tube graft).
<2mm LVA w/ patent RVA can be ligated.

406
Q

A pt w/ aneurysmal arch and descending aorta aneurysm psx w/ chest pain and imaging concerning for contained early rupture (periaortic density). How do you manage?

A

Normally you would want elephant trunk, but this isn’t possible because a distal seal would not occur until the completion elephant trunk.
Do hemi-clamshell incision extended into the L 4th ICS and do full arch and descending aorta surgery.

407
Q

A very elderly patients presents with surgical ascending aneurysm and 4.5 cm proximal arch. Does age affect the extent of surgical resection?

A

Elderly patients do not tolerate circulatory arrest as well as younger patients.
Cannulate the R axillary artery (or go high on the aorta), and do a hemiarch.

408
Q

A patient with known descending TAA comes in with pain, what is the management?

A

Repair.
ALL symptomatic TAAs should be repaired (ruptured, dissected, pain)

409
Q

For descending TAA, what are the asymptomatic indications for repair?

A

For most average-sized adults: Diameter of >5.5 cm. Of note, there isn’t a 5 cm caveat for centers of excellence/experienced surgeons (as opposed to root/ascending).

Patients with high surgical risk (eg, STS Predicted Risk of Mortality [PROM] score >8%, frailty score index >2, two compromised organ systems): Diameter ≥6 to 7 cm.

Genetically mediated conditions: (syndromic or nonsyndromic), a smaller diameter is suggested as an indicator for repair. The specific threshold diameter depends on the condition.

Smaller patients: including many women, a diameter >2x diameter of the nonaneurysmal aorta or the aortic size index can be used.

410
Q

What rate of expansion for TAAs is an indication for repair?

A

Asymptomatic TAA with rapid expansion ≥5 mm per year (or 3 mm x2 yrs) for aneurysms <5 cm in diameter

411
Q

What are LVOT gradients found on echo for HCM? For diagnosis and treatment cutoff.

A

Basal resting gradient: >30 mmHg
Provocative gradients (stress valsalva or amyl nitrite): >30
Intervention is considered when gradient is >50

412
Q

What are you looking for on echo if suspecting HOCM?

A

Level and degree of LVOT obsx.
Degree of SAM of mitral valve.
If cavitary hypertrophy is diffuse (HOCM) or asymmetric/focal (assn AS or HTN).
Gradients: >30 for dx, >50 for intervention.
Septum: 15mm for HOCM, 30mm assd w/ SCD.
Chamber size and fct: athletic hearts can be 13-15mm LV thickness but have normal relaxation and chamber dilation.

413
Q

What can the EKG show in HOCM?

A

LV strain pattern, large T-wave in precordial leads, frequent PVCs

414
Q

When would you do an exercise echo for HOCM? What are you looking for?

A

Some do it for all - risk stratification and assessment for LVOT gradient.

Look for:
Development of symptoms.
LVOT gradient increase.
Failure of BP to increase (as it should w/ exercise).
Arrhythmia.
ST depression.
Mitral regurgitation.

415
Q

For HOCM treatment, what are the pertinent decision-making points?

A

Obstructive physiology. If non-obstructive, no surgery. Need EF to determine meds.

If obstructive physiology, need to know of heart failure symptoms or angina.
At this point, medical mgmt is still first line.

Persistence of symptoms despite medical management will prompt surgical evaluation. The options are surgery (for those who can tolerate) vs alcohol ablation.

DDD pacing is an option for non-surgical patients.

416
Q

A patient w/ HOCM and obstructive features has persistent symptoms despite medical management. He is a poor surgical candidate. What other options are there?

A

Alcohol ablation

417
Q

A patient w/ HOCM and obstructive features has persistent symptoms despite medical management. He is a poor surgical candidate and coronary anatomy is not accessible percutaneously. What other options are there?

A

DDD pacing - reduces the LVOT gradient and SAM, as well as improving MR and NYHA class

https://www.ncbi.nlm.nih.gov/pmc/articles/PMC9225325/

418
Q

You are preparing for a septal myectomy in a HOCM pt. The TEE demonstrates moderate mitral regurgitation. What do you do?

A

Differentiate between primary MR or SAM.
Ask about the relationship between the PML and AML in terms of height, excess tissue, or chordae.
If the PML height is excessive, may need to do repair (sliding valvuloplasty). This should be anticipated preoperatively because a surgeon uncomfortable in this may end up doing a mitral replacement or canceling the case.

419
Q

POD2 after septal myectomy for HOCM, a patient has a harsh systolic murmur and increasing oxygen requirements. PA systolic pressure is increased. What is your differential? What is the workup?

A

If the patient got diuretics, vasodilators, or went into AF, then LV preload will decrease which could precipitate the recurrence of SAM and MR and could explain a similar presentation.

VSD is rare, but should be on the differential, and this is the description.
Get an echo to confirm.
PA cath will also show step up in oxygen sat b/w proximal and distal ports.
Go back to OR, get TEE, and repair the VSD w/ pericardium.

420
Q

How should you treat acute hypotension for HOCM?
What specifically should be avoided?

A

Vasoconstriction (phenylephrine and vasopressin).

Positive inotropy, vasodilators, and diuresis should be avoided - can precipitate MR and SAM.

421
Q

Will septal myectomy affect SAM and MR in HOCM?

A

It will usually correct these pathologies (SAM causes the MR) when there is no other concomitant mitral pathology.

422
Q

Describe AF management perioperative for HOCM.

A

Aggressive as it gets.
AF reduces LV preload and can induce recurrent MR or SAM.
Cardiovert. Beta block. Give fluid. Give vasopressors (phenylephrine).

Psx: harsh systolic murmur after HOCM surgery. You notice AF. Differential would be MR from SAM vs VSD.

423
Q

Describe simply the classic technique for myectomy for HOCM?

A

11 blade incision below the nadir of the RCA and resect 3 cm length of sub aortic muscle.
This can be extended for 7cm to the apex if it is true HOCM.

424
Q

What is the role of genetic testing in family members of HOCM patients?

A

Not used for establishing diagnosis, but only for screening. May predict progression or complications of HOCM.

Counseling patients with HCM regarding the potential for genetic transmission of HCM is one of the cornerstones of care. Screening first-degree family members of patients with HCM, using either genetic testing or an imaging/electrocardiographic surveillance protocol, can begin at any age and can be influenced by specifics of the patient/family history and family preference. As screening recommendations for family members hinge on the pathogenicity of any detected variants, the reported pathogenicity should be reconfirmed every 2 to 3 years.

425
Q

A patient undergoing workup for AVR for severe symptomatic AS is found to have sub-valvular hypertrophy. What should be done?

A

If TEE or cath document non-valvular LVOT w/ subvalvular gradients and normal valve leaflets, septal myectomy can be performed to improve symptoms of heart failure.
Pay attention to TEE when coming off bypass to ensure complete myectomy is done and see if no AS.

426
Q

How do you use an echo to eval cardiac tumors?
What are you looking for?

A

Often diagnostic.
Need to use it to eval size, extent, mobility, stalk, degree of obstruction, chamber and wall involvement, and proximity to valvular/electrical structures.

427
Q

What does cross sectional imaging (CT and MRI) help evaluate in workup of cardiac tumors?

A

Tumor burden, wall penetration, relation to extracardiac structures. Can help eval consistency of mass.

428
Q

What extremity findings might a patient w/ cardiac tumor have? What does it mean?

A

Digital necrosis 2/2 distal embolization.

429
Q

A patient has a LA myxoma. What could cause pulm HTN and obstructive heart failure?

A

Tumor causes diastolic mitral orifice inflow obstruction.

430
Q

What is the surgical goal for patients w/ LA myxoma?

A

Complete excision w/ a 5mm circumferential atrial septal excision with the stalk.

431
Q

If using an LV vent for excision of an LA myxoma, what are the considerations?

A

Ideally should vent through aortic root.
RSPV vent risks tumor embolization.

432
Q

Describe excision of a LA myxoma?

A

Bicaval CPB w/ snares. Antegrade cardioplegia.
Don’t manipulate heart until clamped.
LV vent through aortic root (not R SPV).
Open RIGHT atrium. Visualize tumor stalk on septum, incise 5mm to this and excise the tumor through the septum w/ a 5mm margin. Inspect for any residual tumor and damage to the mitral valve. Use garbage suction here.
Close septum with patch.

Bi-atrial approach may be necessary. Higher risk of arrhythmia.

433
Q

What if during surgery for LA myxoma excision, the patient arrests on induction? What happened? What do you do?

A

Loss of *vasomotor tone -> dec *preload -> tumor obstruction of *MV inflow.
ACLS.
Intubate and give *catecholamines.
Immediate sternotomy, heparin, CPB w/ dual stage via RA vs SVC only vs peripheral.

434
Q

What if when coming off bypass for LA myxoma excision, the patient has an ASD on TEE?

A

Assess size and location. Unless the leak is tiny, go back on bi-caval CPB, find the hole, and fix it (this is why primary closure is not recommended; use a patch).

435
Q

A RA tumor is causing obstructive symptoms, ascites, and hepatomegaly. What are your considerations, workup, and management?

A

Preop assessment of tumor w/ *TEE and *CT/MRI or possibly venography eval the extent and plan for CPB. Depending on extent, the patient may require bypass via the *R IJ or and/or *femoral.

436
Q

What structure must be preserved on the SVC if a surgery requires RA incision extension for an RA tumor?

A

SA node. Make sure to go laterally.
In addition, protect the the tricuspid valve and AV node.

437
Q

A cardiac tumor is high in the SVC and has embolized to the PA. What is your cannulation strategy and operative conduct?

A

Central aortic and peripheral venous - femoral and R IJ. CVL access through contralateral SCV.
Mobilize the SVC high, possibly above the azygous.
Clamp, antegrade, snare. R SPV vent can be used if no L side involvement.
Incise RA and extend high and lateral to avoid SA node.
Inspect RA, RV, TV, and RVOT.
Separate incision can be used on PA to remove embolus.

438
Q

A R side cardiac tumor has embolized to the PA. Intraop it is very distal. What can you do?

A

Brief period of hypothermic arrest may facilitate full removal of the tumor burden from the distal PA.

439
Q

How do you approach ventricular myxomas?

A

Often actually extension from atria below the AV valve.
Can approach via the outflow tract or the AV valve.
If you can do a good inspection, a ventriculotomy is often avoided.

440
Q

You are doing surgery for a myxoma involving the ventricle, and you find that base of the papillary muscle or the chordae of the aortic valve is involved. How do you proceed?

A

Repair or replacement of the valve will be necessary.

441
Q

Small pedunculated mobile density is attached to the aortic valve and is seen during preop echo for CABG. No signs or risk factors for endocarditis. Normal dental exam. What is it?

A

Take note of the exclusionary history and physical taken.
This is likely a *papillary fibroelastoma.
Benign and found incidentally.
*Excise them d/t their *embolic potential. The stalk is usually very small and can be excised *flush with the leaflet without incurring any leaflet damage.
Arise from valvular endocardium.

442
Q

What are the principles of the approach to cardiac tumor excision?

A

Carefully planned *CPB.
Prevention of embolization by *avoiding manipulation before cross clamping and carefully evaluating chambers before closing.
Operative exposure usually through RA, but possibly LA, rarely ventriculotomy.
Septum should be closed w/ *tension free patch.

443
Q

A cancer pt presents with shortness of breath, sinus tachycardia, and hypoxia w/ a clear CXR. What should you be suspicious of?

A

PE

444
Q

What is the diagnostic test of choice for a PE?

A

CT PA

445
Q

What is Virchow’s triad? What are specific examples of each point?

A

Venous stasis: immobility (intubated, spinal injury), bedrest
Endothelial injury: surgery, trauma, access procedures
Hypercoagulable state: malignancy, thrombophilia, oral contraceptives, hx of thrombosis

446
Q

What are some signs of right heart strain on echo (PE pt)?

A

dilation, septal flattening

447
Q

What is the benefit of pulmonary angiography for PE?

A

eval for clot extent and location

448
Q

What designates a PE as high-risk?
How does this change management?
What’s the risk of death?

A

Hemodynamic compromise (shock/hypotension), RV dysfunction.
Tx w/ heparin and thrombolysis vs embolectomy (as opposed to heparin alone).
Risk of imminent death is 10%, 30-day risk is 30%.

449
Q

Absolute contraindications to systemic thrombolytic therapy in acute PE?

A

Intracranial neoplasm, <2 mo intracranial or spinal surgery/trauma, hx of hemorrhagic stroke, active bleeding, bleeding diathesis (trhombocytopenia), nonhemorrhagic stroke w/in 3 mo.

450
Q

A pt w/ PE has RV strain but has contraindication to thrombolysis (CNS neoplasm, hemorrhagic stroke, recent spine trauma/surgery, etc). What options are there?
What if the patient has significant hemodynamic compromise, poor oxygenation, and is a poor surgical candidate?

A

Catheter-embolectomy vs surgical-embolectomy.

If CV and/or resp unstable, consider ECMO.

451
Q

What is the danger to the RV in the setting of PE requiring ECMO?

A

It does not unload the RV. The RV is still pumping some blood against a clot, so there can be myocardial injury until the clot burden is removed.

452
Q

What is the mortality and 1 yr survival for prompt surgical embolectomy in PE pts w/ large clot burden and contraindication to thrombolysis?

A

Operative mortality is 6%.
1-year survival is 86%.
According to 47 pt series by Leacche et al.

453
Q

You take a PE pt w/ HD compromise, large clot, and thrombolysis contraindication to the OR. What do you look for on TEE?

A

RA and RV for clot in transit.
PFO/ASD.
PA for extent of clot.

454
Q

The first step in the management of CTEPH?

A

initiation of anticoagulant therapy, provided the patient is not at a high risk of bleeding, continued indefinitely

455
Q

A patient w/ CTEPH has a high bleeding risk, so there is hesitancy towards anticoagulation. What can be done?

A

inferior vena cava filter until the bleeding risk is resolved and adequate anticoagulation can be provided

456
Q

In order to assess the hemodynamic impairment from CTEPH, what should be done?

A

examine pulmonary artery catheterization (PAC) data:
hemodynamic abnormalities at rest that almost invariably warrant surgery (eg, PVR of 240 to 1200 dynes-sec/cm-5 [3 to 15 Wood units] with the majority falling into the range of 600 to 1000 dynes-sec/cm-5 [7.5 to 12.5 Wood units])

for patients without a substantial increase in pulmonary hemodynamics with exercise, the benefits of PTE are less clear - can consider PTE when the patient’s exercise intolerance and quality of life dissatisfaction is of sufficient magnitude that the patient is willing to accept the risk of surgery

457
Q

How do you determine if chronic thromboemboli is accessible for surgery (CTEPH surgery)?
What if the pt has RV failure?

A

located proximally in the main, lobar, or segmental pulmonary arteries are amenable to surgical removal;
digital subtraction pulmonary angiography (DSPA) is considered the gold standard in defining the extent and location of the thromboembolic obstruction

patients with profound right ventricular failure and shock, the risk of DSPA may be prohibitive to this assessment - do computed tomographic pulmonary angiography (CTPA) or magnetic resonance pulmonary angiography (MRPA)

458
Q

72 hrs after a CTEPH pt undergoes PTE, he has diffuse opacity in RML and RLL lobes. What happened? What do you do?

A

reperfusion syndrome (8-10%); mainly supportive tx w/ vent and pressors prn
diurese and wean FiO2
resolves in 7-10 days

459
Q

A 67M w/ thymoma s/p thymectomy and radiation returns w/ fatigue dyspnea and edema. What is your differential and a possible workup?
What would you predict imaging to show?

A

Chronic radx pericarditis, CHF, restrictive cardiomyopathy, recurrence.
CXR to eval for pericardial calcification or effusions.
CT or MRI may show thickened or calcified pericardium.
EKG is nonspecific w/ ST-T changes.
Echo may show constrictive physiology - bowing of interventricular septum w/ inspiration, thickened pericardium.
Possible cath.

460
Q

What would the echo and cardiac cath show in constrictive pericarditis?

A

Echo - bowing of interventricular septum to the left w/ inspiration (instead of the RV expanding out w/ inc preload, it has to expand into the LV)

Cath - square root sign, RV end diastolic P > 1/3 RV systolic P, equalization of pressures across the atria during diastole, equalization of ventricular pressures during diastole (except during inspiration).

461
Q

What are some distinguishing features of restrictive disease as opposed to constrictive disease?

A

Small or normal sized heart, reduced LV fct, pulmonary and hepatic congestion, BNP may be elevated (unlike in constrictive disease)

462
Q

If diagnosis is in doubt for restrictive CM vs constrictive pericarditis after CXR, CT, echo, cath, and labs, what else can be done?

A

Endomyocardial biopsy or diagnostic surgical exploration with inspection of the pericardium.

463
Q

How do you treat constrictive pericarditis?

A

surgery - pericardial stripping

464
Q

What are the goals of pericardial stripping for constrictive pericarditis?

A

excise pericardium from phrenic to phrenic, down to diaphragm, and around the AV groove to the entrance of the caval/pulmonary veins posteriorly
- CPB is usually needed for at least the left side structures
- there may be a lack of surgical planes, which can lead to significant blood loss and damage to coronaries or venous structures (may want groins exposed, would wait to give heparin until aorta is exposed)

465
Q

While doing posterior dissection during pericardial stripping for constrictive pericarditis, what structure do you have to be wary when moving along the AV groove posteriorly?

A

esophagus - can palpate the TEE probe to stay clear

466
Q

A patient has a history of lymphoma c/b constrictive pericarditis. In the OR, you find large implants not visible on preop imaging. How do you proceed?

A

Send samples for frozen. If malignancy is found, abort and refer to an oncologist.

467
Q

A patient is undergoing pericardial stripping after constrictive pericarditis was dx after his CABG. You injure a bypass vessel in the OR. What do you do?

A

CPB
Check for HD changes and EKG
Repair, interpose, or re-bypass completely.

468
Q

If your descending type B dissection open repair is more proximal near the arch, where would you make the thoracotomy?
What if more distal access is required?

A

3rd or 4th space
7th space if distal access needed

469
Q

What would be considered a “complicated” type B dissection?
Are these more common than uncomplicated?
Why is this category important?

A

Persistent pain, malperfusion (spinal, visceral, extremity), impending rupture, life-threatening complications (very general category…).
Most Type B dissections are uncomplicated.
Complicated dissections usually require surgical or endovascular intervention.

470
Q

During your type B open repair of the descending thoracic aorta, where would you encounter the L RLN?

A

near the ligamentum and along the posterior proximal neck of the aorta

471
Q

During your type B open repair of the descending thoracic aorta, how do you manage the phrenic and vagus nerves on the transverse arch?

A

dissect them free from the transverse arch

472
Q

During your type B open repair of the descending thoracic aorta, what is the benefit of staying above T7?

A

prevent the need for re-implantation of the intercostal arteries and reduce risk of spinal ischemia and thus paraplegia

473
Q

What are the principles of management of dynamic and static branch malperfusion in the context of type B aorta repair?

A

Dynamic malperfusion should be improved after the repair.
Static malperfusion usually requires percutaneous balloon septal fenestration and uncovered stent by IR.

474
Q

How do you manage paraplegia after a type B aorta repair?

A

Lumbar drain.
Ideally ICP less than 10. Do not drain >20ml/hr d/t risk for SDH.
MAP support to 80-90 w/ fluids, phenylephrine, or vasopressin to improve spinal cord perfusion pressure.

No steroids.

475
Q

What are your considerations for a cannulation strategy for type B dissection that is very proximal right off L SCA?

A

Will likely need circ arrest for the proximal portion. May need to reimplant the L SCA separately.
R ax cannulation is ideal.

476
Q

CABG patient gets type A after aortic cannulation confirmed on TEE. You are trying to place a retrograde catheter for plegia, but are unable to after multiple attempts. How do you ensure retrograde plegia placement?

A

Go bicaval, open the atrium, and place the retrograde cannula directly.

477
Q

CABG patient gets type A after aortic cannulation confirmed on TEE. What do you do? What are the operative steps?

A

1) Stop perfusing through the line.
2) Volume and pressors as needed to maintain MAP - can use venous line if you’ve placed one, just make sure it’s deaired.
3) Alternate cannulation - axillary, femoral, percutaneous through ascending aorta confirmed with TEE.
4) Remove the first cannula, assess proximal/distal extent, assess for rupture, AI (LV distention), MI.
5) Retrograde coronary sinus. LV vent (not root vent).
6) Start cooling to 18.
** If heart distends and fibrillates, clamp and arrest.
7) Clamp, arrest retrograde, and vent the root via aortotomy.
**
If AI, do proximal work on valve.
9) Circ arrest, brain perfusion.
10) Hemiarch.
11) Deair, resume full flow.
12) Proximal aorta anastomosis, add root vent.
13) Distal and proximal anastomoses for CABG.

478
Q

You are doing a CABG and begin arterial cannulation for CPB. When would you suspect type A dissection? How do you confirm?

A

Hematoma through the aorta visually. MAP drops. Line pressure is high.
Confirm with TEE. They will see a flap. Determine proximal and distal extent.

479
Q

After CABG, an IABP is placed. In the ICU, heart function recovers, but the leg loses a pulse. The patient is stable. What do you do?

A

Remove the IABP.
If the leg is still without a pulse, check a duplex and CTA to include the chest, abdomen, pelvis, and runoff.
If a type B is found, go to the OR emergently for fem-fem bypass.